Ethics And Professional Issues - Domain Quiz Flashcards

1
Q

Dr. Everett Ernest, a licensed psychologist, has recently started a practice in a small town. He learns from a mutual friend that Dr. Cathy Chatty (also a psychologist) has been revealing confidential information about some of her clients to members of the community. Dr. Ernest should:
Select one:

a. contact the clients to let them know what he has heard.
b. obtain consents from the clients and file a complaint with the Ethics Committee.
c. consult with a colleague about the situation.
d. do nothing since the information about Dr. Chatty is hearsay.

A

Ethical guidelines require psychologists to take some action in situations in which a fellow professional is believed to have acted unethically. However, a psychologist can choose from several alternatives when doing so.

Answer C is correct: Of the answers given, this is the best one. If Dr. Ernest felt comfortable with discussing the issue with Dr. Chatty, that would probably be the best course of action. Since this is not given as an alternative, consulting with a colleague is the best answer.

Answer A is incorrect: Contacting the clients wouldn’t be appropriate.

Answer B is incorrect: Filing a complaint is always an option, but given the way that Dr. Ernest found out about the problem, this would probably not be the best course of action. It also wouldn’t be appropriate to contact Dr. Chatty’s clients.

Answer D is incorrect: It is because the information is “hearsay” that consulting with a fellow professional might be the best initial course of action.

The correct answer is: consult with a colleague about the situation.

How well did you know this?
1
Not at all
2
3
4
5
Perfectly
2
Q

Dr. Marla Martin is asked to provide crisis intervention services to community members who were recently affected by a tornado that destroyed their homes. Dr. Martin does not have experience providing assistance to people who have been traumatized by a natural disaster, but there is no one else in the area who has experience and is available to see these individuals. As an ethical psychologist, Dr. Martin should:
Select one:

a. provide services only if she is supervised by another professional who has experience with victims of a natural disaster.
b. provide services to these individuals but use only interventions that she has experience using.
c. provide services to these individuals but stop when the crisis has ended or other services become available.
d. refuse to provide the services.

A

This issue is addressed in Standard 2.02 of the Ethics Code.

Answer C is correct: Standard 2.02 states: “In emergencies, when psychologists provide services to individuals for whom other mental health services are not available and for which psychologists have not obtained the necessary training, psychologists may provide such services in order to ensure the services are not denied. The services are discontinued as soon as the emergency has ended or appropriate services are available.”

The correct answer is: provide services to these individuals but stop when the crisis has ended or other services become available.

How well did you know this?
1
Not at all
2
3
4
5
Perfectly
3
Q

Dr. Samantha Stern is a psychologist who assists with hiring and promotion decisions at a mental health clinic. She recommends that one of the psychologists NOT be considered for promotion because of an unresolved charge of sexual harassment against him. Dr. Stern’s recommendation is:
Select one:

a. consistent with the provisions of the Ethics Code.
b. a violation of the provisions of the Ethics Code.
c. ethical as long as the clinic considers the psychologist if he is acquitted of the charge.
d. not addressed by the Ethics Code.

A

This situation is explicitly addressed in Standard 1.08 of the APA’s Ethics Code.

Answer B is correct: Standard 1.08 states: “Psychologists do not deny persons employment, advancement, admission to academic or other programs, tenure, or promotion, based solely upon their having made or their being the subject of an ethics complaint.”

The correct answer is: a violation of the provisions of the Ethics Code.

How well did you know this?
1
Not at all
2
3
4
5
Perfectly
4
Q

Dr. Stan Sanchez is not getting along with his neighbor and finds that he has little patience and tends to get angry easily with therapy clients who resemble the man. To be consistent with the requirements of the APA’s Ethics Code, Dr. Sanchez should:
Select one:

a. refer clients who resemble his neighbor to another therapist.
b. inform clients who resemble his neighbor about his current situation and discuss their options with them.
c. consult with another professional to determine the best course of action.
d. monitor his behavior for signs of impaired objectivity and peformance.

A

This situation is addressed in Standard 2.06 of the Ethics Code.

Answer C is correct: Standard 2.06 states that, when psychologists become aware that “personal problems may interfere with their performing work-related duties adequately, they take appropriate measures, such as obtaining professional consultation or assistance.”

Answer A is incorrect: Referral may be appropriate but, before doing so, Dr. Martin would want to discuss this situation with a colleague to determine if that is the best course of action.

Answer B is incorrect: Informing the client about his personal problem would not be an appropriate course of action.

Answer D is incorrect: Monitoring his activity would not be sufficient since Dr. Martin is already aware that his problem with his neighbor is having an adverse impact on his effectiveness with some of his clients.

The correct answer is: consult with another professional to determine the best course of action.

How well did you know this?
1
Not at all
2
3
4
5
Perfectly
5
Q

Pro bono services are ________ by the APA’s Ethics Code.
Select one:

a. required
b. encouraged
c. prohibited
d. not addressed

A

Although the term “pro bono” is not used in the Ethics Code, General Principle B (Fidelity and Responsibility) states that psychologists “strive to contribute a portion of their professional time for little or no compensation or personal advantage.”

Answer B is correct: Since the General Principles are aspirational (rather than mandatory), an implication of the above requirement is that pro bono services are encouraged or recommended – but not required – by the Ethics Code.

The correct answer is: encouraged

How well did you know this?
1
Not at all
2
3
4
5
Perfectly
6
Q

“Vicarious liability” is most likely to be a concern when a psychologist is acting in the role of:
Select one:

a. supervisor.
b. advocate.
c. faculty member.
d. radio talk show host.

A

Under certain circumstances, supervisors may be legally responsible for the actions of their supervisees.

Answer A is correct: The determination of vicarious liability ordinarily rests on the disparity in training and experience between the supervisor and supervisee and the level of responsibility the supervisor has for the supervisee’s actions.

The correct answer is: supervisor.

How well did you know this?
1
Not at all
2
3
4
5
Perfectly
7
Q

You receive a letter from the Ethics Committee asking for information about a former client who has filed a complaint against her current therapist. You stopped seeing the client over seven years ago. You should:
Select one:

a. cooperate with the Committee’s request by sending it a copy of the client’s file.
b. cooperate with the Committee’s request by sending it the information you believe is relevant to the case.
c. cooperate with the Committee’s request by sending it the information you believe is relevant to the case after confirming the client has signed a release.
d. inform the Committee that the information you have about the client is seven years old and that you’re ethically obligated not to forward it to them.

A

Psychologists must cooperate fully with requests from the Ethics Committee.

Answer C is correct: This issue is addressed in Standard 1.06 of the Ethics Code, which notes that, when responding to a request for information from the Ethics Committee, psychologists must “address any confidentiality issues.” Therefore, this answer is most consistent with the requirements of the Ethics Code.

The correct answer is: cooperate with the Committee’s request by sending it the information you believe is relevant to the case after confirming the client has signed a release.

How well did you know this?
1
Not at all
2
3
4
5
Perfectly
8
Q

Your new client is a 37-year-old woman whose symptoms meet the diagnostic criteria for Major Depressive Disorder. After your third session with her, she tells you that she was in treatment with another psychologist but terminated therapy six weeks ago after the psychologist made sexual advances toward her. She does not want you to discuss this matter with anyone. The best course of action in this situation would be to:
Select one:

a. inform the client of the limits of confidentiality in this situation and report the unethical behavior.
b. inform the client of the limits of confidentiality in this situation and contact the other psychologist.
c. discuss the client’s options with her and maintain her confidentiality unless she signs a release.
d. tell the woman that the psychologist’s behavior was unethical and suggest that she file a complaint against him.

A

The APA’s Ethics Code requires psychologists to take action when they learn of an ethical violation by another psychologist but only after considering the client’s confidentiality.

Answer C is correct: This answer is most consistent with the requirements of the Ethics Code and also addresses the best interests of the client – i.e., it is often in the best interests of a client to allow him or her to determine what action should be taken. In addition, Answers A and B can be eliminated because they do not take client confidentiality into consideration, and Answer D is not the best answer because it imposes your preference on the client rather than allowing the client to make her own decision.

The correct answer is: discuss the client’s options with her and maintain her confidentiality unless she signs a release.

How well did you know this?
1
Not at all
2
3
4
5
Perfectly
9
Q

You want to start treating clients who believe they have been abducted by aliens, but you have never worked with members of this population before. Your best course of action would be to:
Select one:

a. start seeing these clients and consult with someone who is familiar with this population if you encounter any problems.
b. start seeing these clients after establishing a consultative relationship with someone who is familiar with this population.
c. take a continuing education course related to the treatment of members of this population before you begin seeing these clients.
d. see these clients only in emergency situations or if no one else is available in the community to see them.

A

This issue is addressed by Standard 2.01 of the Ethics Code.

Answer B is correct: Standard 2.01 requires psychologists to provide services only within the boundaries of their competence and to seek education, training, or supervision when necessary. When working with a new population, the best course of action is often obtaining consultation.

The correct answer is: start seeing these clients after establishing a consultative relationship with someone who is familiar with this population.

How well did you know this?
1
Not at all
2
3
4
5
Perfectly
10
Q

In most situations, the “holder of the privilege” is the:
Select one:

a. client.
b. therapist.
c. client and therapist jointly.
d. court.

A

Privilege is a legal requirement that prohibits (with some exceptions) confidential client information from being disclosed in legal proceedings.

Answer A is correct: In most circumstances, the client is the holder of the privilege, which means that only the client can determine when confidential information may be disclosed in court, a deposition, or other legal proceeding. See the Ethics and Professional Issues chapter in the written study materials for additional information on privilege.
The correct answer is: client.

How well did you know this?
1
Not at all
2
3
4
5
Perfectly
11
Q

Sandy S. is considering filing a claim of sexual harassment against her boss, Bertrand B., because he frequently comments on her physical appearance by telling her how “great she looks” and often tells her that, “if I weren’t your boss, I’d be chasing after you.” Sandy decides to tell Bertrand that she finds his comments offensive and, when she does so, he apologizes and says he’ll stop if that’s what she wants. This situation:
Select one:

a. does not represent a legal claim for sexual harassment because Bertrand’s comments are not sufficiently severe to be considered abusive.
b. does not represent a legal claim for sexual harassment as long as Bertrand actually stops making offensive comments.
c. represents a legal claim for sexual harassment because Sandy found Bertrand’s behavior to be offensive.
d. represents a legal claim for sexual harassment because a “reasonable woman” would find Bertrand’s behavior to be offensive.

A

Sexual harassment laws and policies vary somewhat from jurisdiction to jurisdiction but Bertrand’s comments are likely to be grounds for a sexual harassment suit if he continues to make them.

Answer B is correct: If the comments are not severe and the harasser stops when asked to do so, it is unlikely that they would be found sufficient for a legal claim of sexual harassment. For additional information, see the discussion on sexual harassment in the Ethics and Professional Issues chapter of the written study materials.

Answer A is incorrect: Sexual comments do not have to be “sufficiently severe to be considered abusive” for a claim of sexual harassment.
Answer C is incorrect: Although Bertrand’s comments are likely to be considered offensive, it is unlikely that Sandy would have grounds for a legal claim against him if he stops making those comments when asked to do so.
Answer D is incorrect: If Bertrand does not stop making offensive comments when asked to do so, the court might use the “reasonable woman” standard to determine if the comments represent sexual harassment.

The correct answer is: does not represent a legal claim for sexual harassment as long as Bertrand actually stops making offensive comments.

How well did you know this?
1
Not at all
2
3
4
5
Perfectly
12
Q

The term “privilege” is:
Select one:

a. a general term referring to the right to privacy.
b. an ethical term referring to an obligation not to reveal confidential information.
c. the legal equivalent of the ethical concept of confidentiality.
d. a legal term referring to the protection of confidential information in legal proceedings.

A

For the exam, you want to be familiar with the terms “privilege” and “holder of the privilege.” These are discussed in the chapter on ethics and professional issues in the written study materials.

Answer D is correct: Privilege is a legal concept that protects a client’s confidentiality in the context of a legal proceeding.

The correct answer is: a legal term referring to the protection of confidential information in legal proceedings.

How well did you know this?
1
Not at all
2
3
4
5
Perfectly
13
Q

Which of the following accurately describes the requirements of the APA’s Ethics Code with regard to informed consents?
Select one:

a. An informed consent must be in writing.
b. An informed consent must be in writing and signed by the client.
c. A written or oral consent must be documented.
d. An oral consent is never acceptable.

A

Informed consents are addressed in Standards 3.10, 8.02, 9.03, and 10.01 of the Ethics Code. Additional information about informed consents is provided in the discussion of Standard 3.10 in the Ethics and Professional Issues chapter of the written study materials.

Answer C is correct: Standard 3.10(d) states that “psychologists appropriately document written or oral consent, permission, and assent.”

The correct answer is: A written or oral consent must be documented.

How well did you know this?
1
Not at all
2
3
4
5
Perfectly
14
Q

Which of the following is an example of a psychologist’s obligations with regard to the Tarasoff decision?
Select one:

a. A psychologist has a duty to warn or protect a therapy client’s neighbor when the client says he is planning to murder the neighbor.
b. A psychologist has a duty to protect a therapy client when the client says he is planning to kill himself and has a plan for doing so.
c. A psychologist has a duty to protect a therapy client when the client says his wife has been threatening to kill him and he thinks her last threat was “really serious.”
d. A psychologist has a duty to inform the authorities when a therapy client says he “put someone in the hospital” in a physical fight at a local bar last weekend.

A

For the exam, you want to be familiar with the history and implications of the Tarasoff ruling. This case is described in the Ethics and Professional Issues chapter of the written study materials.

Answer A is correct: The Tarasoff decision established a psychologist’s duty to warn or protect an identifiable third party who is at risk for serious injury from a therapy client.

The correct answer is: A psychologist has a duty to warn or protect a therapy client’s neighbor when the client says he is planning to murder the neighbor.

How well did you know this?
1
Not at all
2
3
4
5
Perfectly
15
Q

While treating a client, you decide that you want to consult with a colleague to discuss a specific aspect of the case that is giving you trouble. According to the Ethics Code:
Select one:

a. you must obtain a release from the client before talking to a consultant.
b. you must obtain a release from the client only if you will be revealing the client’s identity to the consultant.
c. you do not have to obtain a release from the client as long as he or she has already signed a general waiver of confidentiality.
d. you do not have to obtain a release from the client under any circumstances since the consultant is “clearly connected” with the case.

A

This issue is addressed in Standard 4.06 of the Ethics Code.

Answer B is correct: Standard 4.06 states that, “when consulting with colleagues … psychologists do not disclose confidential information that reasonably could lead to the identification of a client or patient … unless they have obtained the prior consent of the person.” It is not, however, necessary to obtain a consent when the client’s identity will not be revealed.

The correct answer is: you must obtain a release from the client only if you will be revealing the client’s identity to the consultant.

How well did you know this?
1
Not at all
2
3
4
5
Perfectly
16
Q

You have been seeing Sal Smythe in individual therapy for several months, and the work-related issues he originally sought therapy for are nearly resolved. During your current session with Sal, he mentions that his 14-year-old son has been getting into trouble in school lately and asks if you would be willing to see the boy in individual therapy. You should:
Select one:

a. agree to see his son in therapy if you have experience working with adolescents.
b. agree to see his son in therapy only after discussing issues related to confidentiality with Sal and his son.
c. agree to see his son in therapy only after determining that the boy’s problems are unrelated to Sal’s issues.
d. tell Sal that you cannot see his son in therapy and provide him with several referrals.

A

Seeing your client’s son in therapy would constitute a multiple relationship unless doing so is for the purpose of continuing to work with Sal on his issues.

Answer D is correct: In most circumstances, you would want to avoid seeing the family members of a client you are seeing in individual therapy since doing so would represent a multiple relationship. No information is given in this question or the answers to suggest that this situation represents an exception to this general rule.

The correct answer is: tell Sal that you cannot see his son in therapy and provide him with several referrals.

How well did you know this?
1
Not at all
2
3
4
5
Perfectly
17
Q

Your new clients are a family that consists of a husband, wife, and two adolescent girls. They began therapy because one of the girls has started using drugs and the other has signs of an eating disorder. You realize during the first session that you and the parents belong to the same country club. You should:
Select one:

a. terminate therapy with this family and provide them with referrals.
b. ask the parents to decide if they want to continue therapy with you.
c. consider the situation to determine if you can maintain your objectivity with this family.
d. continue seeing the family and join a different country club.

A

This question describes a multiple relationship.

Answer C is correct: Multiple relationships are addressed in several Standards of the Ethics Code, including Standard 3.05, which prohibits multiple relationships that “could be reasonably expected to impair the psychologist’s objectivity, competence, or effectiveness … otherwise risks exploitation or harm to the person.” In this case, you would want to determine if the parents’ membership in the country club is likely to have an adverse effect on your effectiveness before proceeding with treatment. Additional information on multiple relationships is provided in the Ethics and Professional Issues chapter of the written study materials.

The correct answer is: consider the situation to determine if you can maintain your objectivity with this family.

How well did you know this?
1
Not at all
2
3
4
5
Perfectly
18
Q

According to the APA’s Ethics Code, psychologists should discuss fees and other financial arrangements with clients:
Select one:

a. during the initial phone contact.
b. during the first therapy session.
c. as early as is feasible.
d. at the psychologist’s discretion.

A

Fees and other financial arrangements are addressed in Standard 6.0 of the Ethics Code.

Answer C is correct: This answer is the best one of those given because it contains the exact language of Standards 6.04 and 10.01, which both address the discussion of fees with clients.

The correct answer is: as early as is feasible.

How well did you know this?
1
Not at all
2
3
4
5
Perfectly
19
Q

A client’s insurance company pays 75% of a client’s therapy fee, while the client is required to pay the remaining 25%. The client asks you to bill the insurance company at a higher hourly rate so that he will not have to pay his 25% co-payment. If you do so, you will be acting:
Select one:

a. illegally and unethically.
b. illegally but ethically.
c. legally but unethically.
d. legally and ethically.

A

Billing the insurance company at a higher rate so that the client does not have to make his co-payment would be insurance fraud (unless, of course, the insurance company has approved this arrangement which it is not likely to do).

Answer A is correct: Misrepresentation of your fee to the insurance company represents fraud and is, therefore, both illegal and unethical.

The correct answer is: illegally and unethically.

How well did you know this?
1
Not at all
2
3
4
5
Perfectly
20
Q

A client you had been seeing in therapy for two months suddenly stopped coming to her sessions. You have been unable to contact her and she owes you for the last three sessions. You are thinking about using a collection agency to collect her outstanding fees. In terms of the requirements of the APA’s Ethics Code, the use of a collection agency is:
Select one:

a. always unacceptable.
b. acceptable only as a “last resort.”
c. acceptable as long as you discussed your policy with her at the beginning of therapy and she agreed to it.
d. acceptable as long as you inform her of your intent and give her an opportunity to pay her outstanding fee before doing so.

A

The use of a collection agency to obtain outstanding fees is addressed in Standard 6.04(e) of the Ethics Code.

Answer D is correct: This answer is most consistent with Standard 6.04(e), which states that, “if the recipient of services does not pay for services as agreed, and if psychologists intend to use collection agencies or legal measures to collect the fees, psychologists first inform the person that such measures will be taken and provide that person an opportunity to make prompt payment.”

The correct answer is: acceptable as long as you inform her of your intent and give her an opportunity to pay her outstanding fee before doing so.

How well did you know this?
1
Not at all
2
3
4
5
Perfectly
21
Q

Dr. Terry Tawlk includes two testimonials in a brochure describing his weekend communications workshop for couples. The first is from a colleague and states, “Communication is a key to a happy marriage and Dr. Tawlk is one of the leading experts on communication. If you feel you can’t talk to your partner, I’d recommend Dr. Tawlk’s workshop. It’s likely to have a positive effect on your relationship.” The second testimonial is from a former workshop participant who said Dr. Tawlk could use an excerpt from a letter she sent him following the workshop that she and her husband attended. It states, “It worked! I can now tell my husband what I need without it turning into a major argument!” Which of the following is true about the testimonials included in Dr. Tawlk’s brochure?
Select one:

a. They are both unethical since testimonials are prohibited under any circumstances.
b. The testimonial from the colleague is unethical but the testimonial from the former client is acceptable.
c. The testimonial from the former client is unethical but the testimonial from the colleague is acceptable.
d. The testimonials do not seem to violate the provisions of the Ethics Code and are acceptable.

A

Standard 5.05 of the Ethics Code does not prohibit the use of testimonials but prohibits psychologists from soliciting them from “current therapy clients or patients or other persons who … are vulnerable to undue influence.”

Answer D is correct: The testimonials described in this question do not violate the provisions of Standard 5.05. In addition, they do not appear to be false, deceptive, or misleading and, therefore, do not violate other requirements of the Ethics Code.

The correct answer is: The testimonials do not seem to violate the provisions of the Ethics Code and are acceptable.

How well did you know this?
1
Not at all
2
3
4
5
Perfectly
22
Q

When attempting to expand your private practice, it is important to keep in mind that in-person solicitation of business:
Select one:

a. is unethical in all circumstances.
b. is unethical only when the solicitation includes misleading or coercive information.
c. is ethical only when the person being solicited is not receiving similar services from another professional.
d. may be ethical if the person being solicited is not susceptible to undue influence.

A

This issue is addressed in Standard 5.06 of the APA’s Ethics Code.

Answer D is correct: Standard 5.06 states that “psychologists do not engage, directly or through agents, in uninvited in-person solicitation of business from actual or potential therapy clients or patients or other persons who because of their particular circumstances are vulnerable to undue influence. However, this does not preclude (1) attempting to implement appropriate collateral contacts for the purpose of benefiting an already engaged therapy client or patient or (2) providing disaster or community outreach services.” Of the answers given, this one is most consistent with Standard 5.06.

The correct answer is: may be ethical if the person being solicited is not susceptible to undue influence.

How well did you know this?
1
Not at all
2
3
4
5
Perfectly
23
Q

Which of the following best describes the requirements of the APA’s Ethics Code for bartering?
Select one:

a. Bartering for goods (but not services) is acceptable.
b. Bartering for services (but not goods) is acceptable.
c. Bartering for goods or services may be acceptable under certain conditions.
d. Bartering for goods or services is prohibited in all circumstances.

A

Bartering is addressed in Standard 6.05 of the Ethics Code.

Answer C is correct: Standard 6.05 states that “psychologists may barter only if (1) it is not clinically contraindicated, and (2) the resulting arrangement is not exploitative.” For additional information about bartering, see the discussion of Standard 6.05 in the Ethical and Professional Issues chapter of the written study materials.

The correct answer is: Bartering for goods or services may be acceptable under certain conditions.

How well did you know this?
1
Not at all
2
3
4
5
Perfectly
24
Q

You have made an arrangement with another psychologist to exchange a nominal payment of $25.00 for referrals to each other. In terms of the requirements of the APA’s Ethics Code, this is:
Select one:

a. ethical since the Ethics Code does not prohibit referral fees.
b. ethical as long as the clients are informed of this practice at the outset of therapy.
c. unethical if the payment of $25.00 is based on the referral only.
d. unethical if the payment for the referral represents more than 25% of the fee paid by the client.

A

Referral fees are addressed in Standard 6.07 of the Ethics Code.

Answer C is correct: Standard 6.07 states: “When psychologists pay, receive payment from, or divide fees with another professional … the payment to each is based on the services provided (clinical, consultative, administrative, or other) and is not based on the referral itself.”

The correct answer is: unethical if the payment of $25.00 is based on the referral only.

How well did you know this?
1
Not at all
2
3
4
5
Perfectly
25
Q

Your former client, Caleb Cash, has failed to pay the balance of an unpaid bill. Although you have given Caleb numerous opportunities to respond to your written requests for payment and have informed him of your intention to turn this matter over to a collection agency, he has not made any attempt to resolve this matter. One day, you receive a call from another therapist who is currently working with Caleb. She asks you to forward a copy of Caleb’s records and faxes you a copy of a signed release from Caleb. According to the APA’s Ethics Code, you:
Select one:

a. must comply with the therapist’s request since Caleb has signed a release.
b. may withhold Caleb’s records until he pays his outstanding bill.
c. may withhold Caleb’s records as long as they are not needed for emergency treatment.
d. should explain the situation to the therapist and ask her to tell Caleb to contact you.

A

This issue is addressed in Standard 6.03 of the APA’s Ethics Code.

Answer C is correct: Standard 6.03 states that “psychologists may not withhold records under their control that are requested and needed for client’s or patient’s emergency treatment solely because payment has not been received.”

The correct answer is: may withhold Caleb’s records as long as they are not needed for emergency treatment.

How well did you know this?
1
Not at all
2
3
4
5
Perfectly
26
Q
A psychologist sets up a private practice in a lower middle-class urban neighborhood. After several months, he realizes that his lack of clients is due primarily to the fact that most of the residents of the neighborhood cannot afford his hourly fee. He therefore decides that he will include in future advertisements a statement indicating that his services are provided on a "sliding scale." This practice is:
Select one:

a. clearly unethical since sliding scale fees are prohibited by ethical guidelines.
b. clearly unethical since including information on fees in advertisements is prohibited by ethical guidelines.
c. clearly unethical since advertising professional services is prohibited by ethical guidelines.
d. in accord with ethical guidelines.

A

This question addresses two issues - advertising and fees.

Answer D is correct: Sliding scale fees are ethically acceptable as long as fee arrangements are made clear from the onset of treatment and the fee policy is fair and non-exploitative. Advertising one’s professional services and fee policy are also acceptable.

The correct answer is: in accord with ethical guidelines.

How well did you know this?
1
Not at all
2
3
4
5
Perfectly
27
Q

A graduate student designed and conducted a research project for her dissertation and subsequently wrote a journal article describing the study and its results. The original idea for the study was derived from the work of the student’s faculty advisor who also provided the facilities for the student’s research and provided the student with some guidance while she conducted her study. The faculty advisor wants to be listed as first author on the article. In this situation:
Select one:

a. the student should be listed as the sole author.
b. the student should be listed as the first author and the advisor as the second author.
c. it is up to the advisor to determine authorship credit.
d. it is up to the journal to determine authorship credit.

A

Publication credit is addressed in Standard 8.12 of the Ethics Code.

Answer B is correct: Standard 8.12 states that publication credits “accurately reflect the relative scientific or professional contributions of the individuals involved, regardless of their relative status…. Except under exceptional circumstances, a student is listed as principal author on any multiple-authored article that is substantially based on the student’s doctoral dissertation.” While the student should be listed as first author, the advisor’s contribution appears to be sufficient to warrant being listed as the second author.

The correct answer is: the student should be listed as the first author and the advisor as the second author.

How well did you know this?
1
Not at all
2
3
4
5
Perfectly
28
Q

Dr. Betty Black’s ad for her seminar on relaxation techniques invites the public to “come and hear an in-depth lecture on techniques to help reduce the stress of daily living.” The advertised cost for her two-hour seminar is $35.00. While preparing her lecture the night before the presentation, Dr. Black decides it would be valuable to demonstrate her techniques with the group. She decides that, instead of lecturing for the full two hours, she will have attendees divide into groups for the second hour to experiment with the techniques. With regard to the requirements of the APA’s Ethics Code, this is:
Select one:

a. ethical if she allows participants to leave after the lecture portion of the seminar.
b. ethical if the demonstration is well executed and educational.
c. unethical since she advertised the seminar as a “lecture.”
d. unethical because she plans to spend one hour lecturing when the most beneficial part will probably be the experiential portion.

A

This issue is addressed in Standard 7.02 of the Ethics Code.

Answer C is correct: Standard 7.02 requires psychologists to “take reasonable steps to ensure that there is a current and accurate description of the program content.” Altering the program content on the night before its presentation would violate this requirement.

The correct answer is: unethical since she advertised the seminar as a “lecture.”

How well did you know this?
1
Not at all
2
3
4
5
Perfectly
29
Q
Dr. Billie Bloom requires students in his introductory psychology class to participate in a university-sponsored research project as a requirement for the course. This is:
Select one:

a. unethical under any circumstances.
b. ethical as long as the resarch exposes students to “minimum risk.”
c. ethical as long as students are given a choice of completing an alternative assignment.
d. ethical as long as students are made aware of this requirement before they enroll in his class.

A

This issue is addressed in Standard 8.04(b) of the APA’s Ethics Code.

Answer C is correct: Standard 8.04(b) states that, “when research participation is a course requirement or an opportunity for extra credit, the prospective participant is given the choice of equitable alternative activities.”

The correct answer is: ethical as long as students are given a choice of completing an alternative assignment.

How well did you know this?
1
Not at all
2
3
4
5
Perfectly
30
Q

Dr. Cecelia Cooper, a licensed psychologist, is planning a longitudinal study to identify environmental contributors to drug abuse. Her study will involve interviewing drug and non-drug users and their families several times over a five-year period and will require that she maintain participants’ names, addresses, and phone numbers in her files. Dr. Cooper is concerned about the confidentiality of her participants, especially if she is ever called to testify about any of them in court. Dr. Cooper:
Select one:

a. need not be concerned about confidentiality since it will be both legal and ethical for her to refuse to testify about the participants in court.
b. should not be concerned about confidentiality since the participants’ privilege will be waived in this situation.
c. should inform her participants of the possible limits of confidentiality as part of the informed consent process.
d. should always obtain an informed consent from a participant prior to testifying about him or her in court.

A

Confidentiality of research participants is addressed by Standard 8.02 of the APA’s Ethics Code and Principle I.24 of the Canadian Code of Ethics.

Answer C is correct: This answer is most consistent with the provisions of Standard 8.02(a) of the Ethics Code and Principle I.24 of the Canadian Code of Ethics, which stress the importance of informing research participants of potential limits to confidentiality during the informed consent process.

Answer D is incorrect: This answer is incorrect because it refers to an informed consent rather than to a waiver (release) of confidentiality. In addition, a waiver may not be necessary if the situation is one in which privilege is waived.

The correct answer is: should inform her participants of the possible limits of confidentiality as part of the informed consent process.

How well did you know this?
1
Not at all
2
3
4
5
Perfectly
31
Q

Which of the following best describes the provisions of the Ethics Code with regard to sexual relationships with students?
Select one:

a. The Code prohibits relationships with current or former students under any circumstances.
b. The Code prohibits relationships with current students but permits relationships with past students.
c. The Code prohibits relationships with students over whom a psychologist has or is likely to have evaluative authority.
d. The Code prohibits relationships with current or past students who are vulnerable to undue influence.

A

Sexual relationships with students and supervisees are addressed in Standard 7.07 of the APA’s Ethics Code.

Answer C is correct: This answer is most consistent with the actual language of Standard 7.07, which states: “Psychologists do not engage in sexual relationships with students or supervisees who are in their department, agency, or training center or over whom psychologists have or are likely to have evaluative authority.”

The correct answer is: The Code prohibits relationships with students over whom a psychologist has or is likely to have evaluative authority.

How well did you know this?
1
Not at all
2
3
4
5
Perfectly
32
Q

Which of the following best describes the requirements of the APA’s Ethics Code with regard to the use of animals in research?
Select one:

a. Animals should never be used as research subjects.
b. Animals may be used as research subjects only when an alternative procedure is unavailable and the goal of the study is justified by its prospective value.
c. Animals may be used as research subjects only when the procedures used will not cause them pain or stress.
d. Animals may be used as research subjects when they will be subjected to pain or stress but only when an alternative procedure is not available and the goal of the study is justified by its prospective value.

A

The use of animals in research is addressed in Standard 8.09 of the Ethics Code.

Answer D is correct: The Ethics Code permits the use of animals in research under certain conditions, and this answer best describes those conditions.

The correct answer is: Animals may be used as research subjects when they will be subjected to pain or stress but only when an alternative procedure is not available and the goal of the study is justified by its prospective value.

How well did you know this?
1
Not at all
2
3
4
5
Perfectly
33
Q

You have been hired as a research assistant for a study investigating the effectiveness of various forms of therapy for Social Anxiety Disorder. The procedures and methods used in the study are ethically sound; however, your supervisor is not informing participants that they can withdraw from the study at any time if they desire to do so. Your best course of action would be to:
Select one:

a. quit your job.
b. inform the participants yourself of their right to withdraw.
c. discuss the possible ethical violation with the supervisor.
d. do nothing since this is an acceptable practice.

A

Guidelines for conducting research are provided in Standard 8 of the APA’s Ethics Code.

Answer C is correct: In most circumstances, it is necessary to obtain an informed consent from research participants and one requirements for an informed consent is letting potential participants know that they can withdraw from the study at any time. Discussing this issue with the supervisor is consistent with Standard 1.04 (Informal Resolution of Ethical Violations) and would be the best initial course of action.

The correct answer is: discuss the possible ethical violation with the supervisor.

How well did you know this?
1
Not at all
2
3
4
5
Perfectly
34
Q

You have just completed a research study but have not yet published its results. A colleague of yours who is familiar with your study requests the data you have collected. Which of the following best describes the requirements of the APA’s Ethics Code with regard to this situation?
Select one:

a. You are required to comply with the colleague’s request only if her purpose in requesting the data is to re-analyze it.
b. You are required to comply with the colleague’s request only if she agrees to protect the confidentiality of the study’s participants.
c. You are not required to comply with the colleague’s request since the results of the study have not yet been published.
d. You are not required to comply with the colleague’s request unless she has a “legitimate concern” about the accuracy the data.

A

This issue is addressed in Standard 8.14 of the APA’s Ethics Code.

Answer C is correct: Standard 8.14 states the following: “After research results are published, psychologists do not withhold the data on which their conclusions are based from other competent professionals who seek to verify the substantive claims through reanalysis and who intend to use such data only for that purpose, provided that the confidentiality of the participants can be protected and unless legal rights concerning proprietary data preclude their release.” Since you have not yet published your study, you are not obligated to comply with the colleague’s request.

The correct answer is: You are not required to comply with the colleague’s request since the results of the study have not yet been published.

How well did you know this?
1
Not at all
2
3
4
5
Perfectly
35
Q

A 40-year-old therapist becomes attracted to a 38-year-old client and soon realizes that the feelings are mutual. They discuss the situation and agree to terminate therapy and begin dating immediately. This is:
Select one:

a. ethical since they terminated therapy before beginning to date.
b. ethical since the client is an adult and voluntarily agreed to stop therapy.
c. unethical because the therapist began dating a former client immediately after terminating therapy.
d. unethical unless the therapist and client received counseling related to this issue when they began dating.

A

Sexual relationships with former therapy clients are addressed in Standard 10.08 of the APA’s Ethics Code, and additional information about this issue is provided in the Ethics and Professional Issues chapter of the written study materials.

Answer C is correct: Standard 10.08 prohibits psychologists from becoming sexually involved with former therapy clients within two years of terminating the professional relationship and, even then, the relationship may be acceptable only in “the most unusual circumstances.”

The correct answer is: unethical because the therapist began dating a former client immediately after terminating therapy.

How well did you know this?
1
Not at all
2
3
4
5
Perfectly
36
Q

A client you have been seeing in therapy for seven weeks tells you that her ex-husband is threatening to kill you because he blames you for her unwillingness to get back together with him. From your conversations with the client, you know that the man has a history of violent behavior and that his threat against you must be taken seriously. As an ethical psychologist:
Select one:

a. you must continue seeing the client and should contact the police to obtain protection only with the client’s consent to do so.
b. you must continue seeing the client but may contact the police to obtain protection with or without the client’s consent to do so.
c. you may terminate therapy with the client and may contact the police to obtain protection with or without the client’s consent to do so.
d. you may terminate therapy with the client but should contact the police to obtain protection only with the client’s consent to do so.

A

Terminating therapy with a client in this situation is addressed in Standard 10.10(b) of the APA’s Ethics Code.

Answer C is correct: Standard 10.10(b) states that “psychologists may terminate therapy when threatened or otherwise endangered by the client or patient or another person with whom the client or patient has a relationship.” In addition, prohibitions against breaching client confidentiality do not apply if the psychologist needs to contact the police or others to obtain protection for him or herself (see, e.g., C. B. Fisher, Decoding the Ethics Code: A practical guide for psychologists, Thousand Oaks, CA, Sage Publications, 2003).

The correct answer is: you may terminate therapy with the client and may contact the police to obtain protection with or without the client’s consent to do so.

How well did you know this?
1
Not at all
2
3
4
5
Perfectly
37
Q

As defined in the APA’s Ethics Code, a psychologist would be violating test security when he:
Select one:

a. shows a client the questions from a standardized test when discussing the results of the test with the client.
b. discusses a client’s responses to several test items to help the client understand why she received a low score.
c. provides a client with the exact scale and subscale scores she obtained on a standardized test.
d. engages in any of the activities described in the above answers.

A

Test security is addressed in Standard 9.11 of the APA’s Ethics Code.

Answer A is correct: Standard 9.11 defines “test materials” as manuals, instruments, protocols, and test questions and states that “psychologists make reasonable efforts to maintain the integrity and security of test materials and other assessment techniques.” The Code also distinguishes between test materials and test data, which include raw and scaled scores, client responses to test items, and the psychologist’s notes about the client’s responses and behaviors during testing. In most circumstances, providing test data to a client is acceptable and does not represent a breach of test security.

The correct answer is: shows a client the questions from a standardized test when discussing the results of the test with the client.

How well did you know this?
1
Not at all
2
3
4
5
Perfectly
38
Q

Dr. Lisa Lopez has been treating Glenn G. for depression for three weeks. During their most recent session, Glenn disclosed that he has also been seeing another therapist for several months. As an ethical psychologist, Dr. Lopez should:
Select one:

a. tell Glenn he cannot continue seeing him in therapy since he is receiving services from another professional.
b. tell Glenn that he will need to decide which therapist he wishes to continue seeing with the next few weeks.
c. discuss this issue with Glenn and consult with the other therapist after getting Glenn’s approval to do so.
d. continue therapy with Glenn since he has not expressed a desire to stop.

A

Standard 10.04 of the APA’s Ethics Code applies to this situation.

Answer C is correct: This answer is most consistent with the requirements of Standard 10.04. Dr. Lopez should discuss the issue with Glenn, determine if there is a duplication of services (e.g., Glenn may be seeing a behavioral therapist who is helping him stop smoking), and, if appropriate, consult with the other therapist after obtaining Glenn’s permission to do so. Of course, if the two therapists are providing duplicate services, Glenn will have to eventually choose between them, but that is not the best initial action.

The correct answer is: discuss this issue with Glenn and consult with the other therapist after getting Glenn’s approval to do so.

How well did you know this?
1
Not at all
2
3
4
5
Perfectly
39
Q

Dr. Blitz has been seeing a client for four months. The client mentioned that he is very dissatisfied with the progress of therapy in the last two sessions. Dr. Blitz should:
Select one:

a. tell the client that he can’t expect major changes in only four months.
b. encourage the client to schedule more frequent sessions.
c. help the client work through his resistance.
d. discuss the matter with the client and provide him with referrals if he desires.

A

Standard 10.10(a) of the APA’s Ethics Code applies to this situation.

Answer D is correct: Standard 10.10(a) states that psychologists must terminate therapy when it is reasonably clear that the client is no longer benefiting from it. Even though four months is not a long duration for therapy in many situations, the client should not be pressured to continue.

The correct answer is: discuss the matter with the client and provide him with referrals if he desires.

How well did you know this?
1
Not at all
2
3
4
5
Perfectly
40
Q

Dr. Rita Ryan is a psychotherapist in a small town. She receives a phone call from a man she was sexually involved with several years ago. The relationship lasted for only four months and ended amicably. The man is now married and is having problems unrelated to his relationship with Dr. Ryan. He has called to see if he can begin seeing her in therapy. Dr. Ryan should:
Select one:

a. see the man in therapy only if she determines that their past relationship will not impair her objectivity.
b. see the man only if their relationship ended more than two years ago and she determines that it will not impair her objectivity.
c. see the man but discuss the potential for conflicts with him before beginning treatment.
d. refer the man to a colleague.

A

This situation is addressed by Standard 10.07 of the APA’s Ethics Code.

Answer D is correct: Standard 10.07 states that “psychologists do not accept as therapy clients or patients persons with whom they have engaged in sexual intimacies.” There is no time limit on this prohibition.

The correct answer is: refer the man to a colleague.

How well did you know this?
1
Not at all
2
3
4
5
Perfectly
41
Q

Part of your job at a mental health clinic is to administer intelligence, personality, and other tests to child, adolescent, and adult clients. To reduce your workload, you train an employee of the clinic to administer many of the tests and to write interpretive reports. The employee is one semester away from a B.A. in psychology and has already been accepted into a Psy.D. program for the following year. Which of the following is true about this situation?
Select one:

a. You are acting ethically since it is up to you to decide if the employee is sufficiently competent to administer tests and interpret their results.
b. You are acting ethically as long as you supervise the employee’s work.
c. You are acting unethically because only people who have completed some graduate-level work should administer psychological tests.
d. You are acting unethically because the employee is not qualified to interpret many commonly-used psychological tests.

A

Standard 9.07 of the APA’s Ethics Code prohibits psychologists from promoting the use of tests by unqualified persons unless it is being done for training purposes under appropriate supervision.

Answer D is correct: A person who is working on a B.A. in psychology may be qualified to administer some psychological tests but is clearly not qualified to interpret their results. Although it is true that it is up to the psychologist to decide when an employee is qualified to administer tests, in this case, this employee is clearly not qualified to interpret them.

The correct answer is: You are acting unethically because the employee is not qualified to interpret many commonly-used psychological tests.

How well did you know this?
1
Not at all
2
3
4
5
Perfectly
42
Q

Which of the following best describes a psychologist’s ethical requirements with regard to the use of computer-assisted psychological test scoring and interpretation services?
Select one:

a. These services are acceptable but the psychologist is responsible for their appropriate application and use.
b. These services are acceptable and the publisher is responsible for ensuring they provide accurate information.
c. These services are acceptable only when the interpretations are based on objective data.
d. These services are acceptable only in the “most unusual circumstances.”

A

The use of computer-assisted scoring and interpretation services is addressed in Standard 9.09 of the APA’s Ethics Code.

Answer A is correct: This Standard states: “(b) Psychologists select scoring and interpretation services (including automated services) on the basis of the validity of the program and procedures as well as on other appropriate considerations…. (c) Psychologists retain responsibility for the appropriate application, interpretation, and use of assessment instruments, whether they score and interpret such tests themselves or use automated or other services.”

The correct answer is: These services are acceptable but the psychologist is responsible for their appropriate application and use.

How well did you know this?
1
Not at all
2
3
4
5
Perfectly
43
Q

You have been seeing Alonzo Alvarez in therapy for over six months and believe he is no longer benefitting from his sessions with you. However, when you discuss the possibility of ending therapy, he says he’s very happy with the progress he is making and wants to continue. As an ethical psychologist, you:
Select one:

a. agree to continue seeing Alonzo since he is satisfied with the progress of therapy.
b. agree to continue seeing Alonzo for a specified period of time but, at the end of that period, insist on ending therapy.
c. explain to Alonzo that these are normal reactions to the end of therapy and set a date for termination.
d. see if additional goals for treatment can be identified but, if not, set a date for termination and provide Alonzo with referrals if appropriate.

A

Termination of therapy is addressed in Standard 10.10 of the Ethics Code.

Answer D is correct: Standard 10.10 states that “psychologists terminate therapy when it becomes reasonably clear that the client or patient no longer needs the service, is not likely to benefit, or is being harmed by continued service…. Except where precluded by the actions of clients or patients or third-party payors, prior to termination psychologists provide pretermination counseling and suggest alternative service providers as appropriate.”

The correct answer is: see if additional goals for treatment can be identified but, if not, set a date for termination and provide Alonzo with referrals if appropriate.

How well did you know this?
1
Not at all
2
3
4
5
Perfectly
44
Q

A couple who recently moved to the United States from Mexico brings their 9-year-old son for assessment to determine if he meets the diagnostic criteria for ADHD. The child speaks very little English but his mother speaks both Spanish and English fluently. You do not speak Spanish and are unable to find an interpreter for the boy. You should:
Select one:

a. refuse to conduct the assessment and provide the family with appropriate referrals.
b. conduct the assessment using only informal measures that do not depend on language.
c. conduct the assessment but explain the limitations of the results in your report.
d. conduct the assessment with the mother acting as the interpreter.

A

In this situation, you are not qualified to conduct the assessment since you cannot communicate directly with the child in Spanish and are unable to find an interpreter.

Answer A is correct: This issue is addressed directly and indirectly in several APA documents. For example, the Guidelines for Providers of Psychological Services to Ethnic, Linguistic, and Culturally Diverse Populations states that, when the linguistic skills of the psychologist do not match those of the client, the psychologist obtains an appropriate translator or refers the client to another professional.

Answer B is incorrect: Using only measures that do not depend on language would not be the best course of action since doing so would limit your ability to conduct a thorough evaluation.

Answer C is incorrect: Explaining the limitations in the report would not be adequate in this situation.

Answer D is incorrect: The Guidelines also state that, when translation is necessary, psychologists do not retain the services of a translator who has a dual role with the individual in order to avoid jeopardizing the validity of the assessment.

The correct answer is: refuse to conduct the assessment and provide the family with appropriate referrals.

How well did you know this?
1
Not at all
2
3
4
5
Perfectly
45
Q

An organizational psychologist with a Ph.D. wants to become a clinical psychologist. According to APA guidelines, he must:
Select one:

a. complete appropriate coursework.
b. complete appropriate coursework and clinical supervision.
c. obtain supervision from a licensed psychologist.
d. obtain a Ph.D. or Psy.D. in clinical psychology from an accredited school.

A

This issue is addressed in Section 1.7 of the APA’s General Guidelines for Providers of Psychological Services.

Answer B is correct: Section 1.7 states that “psychologists who change or add a specialty meet the same requirements with respect to subject matter and professional skills that apply to doctoral education, training, and experience in the new specialty.” In other words, the psychologist would have to complete relevant courses and requirements for supervised practice.

The correct answer is: complete appropriate coursework and clinical supervision.

How well did you know this?
1
Not at all
2
3
4
5
Perfectly
46
Q

Dr. Paula Pang is hired to perform a court-ordered evaluation of a defendant in a criminal case. After describing the purpose of the evaluation to the defendant, he says that he is not willing to cooperate. According to the APA’s Specialty Guidelines for Forensic Psychology, Dr. Pang should:
Select one:

a. remind the defendant that he has no choice since the evaluation is court-ordered.
b. refuse to conduct the evaluation unless she obtains a signed consent from the defendant’s attorney.
c. obtain as much information from the defendant as possible and, in her report, explain why it is incomplete.
d. postpone the evaluation and inform the defendant that he should discuss this matter with his attorney.

A

This situation is addressed in the Specialty Guidelines for Forensic Psychology (APA, 2012).

Answer D is correct: Paragraphs 6.03.02 of the Specialty Guidelines states: “If the examinee is ordered by the court to participate, the forensic practitioner can conduct the examination over the objection, and without the consent, of the examinee…. If the examinee declines to proceed after being notified of the nature and purpose of the forensic examination, the forensic practitioner may consider a variety of options including postponing the examination, advising the examinee to contact his or her attorney, and notifying the retaining party about the examinee’s unwillingness to proceed.”

The correct answer is: postpone the evaluation and inform the defendant that he should discuss this matter with his attorney.

How well did you know this?
1
Not at all
2
3
4
5
Perfectly
47
Q

During a court-ordered evaluation of a defendant to determine her competence to stand trial, she reveals information to you that confirms her guilt. As an ethical psychologist, you should:
Select one:

a. use your discretion in determining what to include in your evaluation report.
b. include information about the defendant’s confession in the report since privilege is waived in this situation.
c. include only information relevant to the defendant’s competence in the evaluation report.
d. not provide the court with any information obtained in the evaluation without a release from the defendant.

A

This situation is addressed in the Specialty Guidelines for Forensic Psychology (APA, 2012).

Answer C is correct: This answer is most consistent Paragraph 10.01 of the Specialty Guidelines. It states: “Forensic examiners seek to assist the trier of fact to understand evidence or determine a fact in issue, and they provide information that is most relevant to the psycholegal issue. In reports and testimony forensic practitioners typically provide information about examinees’ functional abilities, capacities, knowledge, and beliefs, and ADDRESS THIER OPINIONS AND RECOMMENDATIONS TO THE IDENTIFIED PSYCHOLEGAL ISSUES.”

The correct answer is: include only information relevant to the defendant’s competence in the evaluation report.

How well did you know this?
1
Not at all
2
3
4
5
Perfectly
48
Q

When hired as an expert witness by the attorney for a plaintiff in a criminal case, a psychologist:
Select one:

a. may provide his or her services on the basis of contingent fees only if permitted to do so by the court.
b. may provide his or her services on the basis of contingent fees only when doing so does not interfere with his or her objectivity.
c. may provide his or her services on the basis of contingent fees as long the plaintiff consents to this arrangement.
d. should ordinarily avoid providing his or her services on the basis of contingent fees.

A

This issue is addressed in Paragraph 5.02 of APA’s Specialty Guidelines for Forensic Psychology (APA, 2012).

Answer D is correct: This answer is most consistent with Paragraph 5.02, which states: “Forensic practitioners seek to avoid undue influence that might result from financial compensation or other gains. Because of the threat to impartiality presented by the acceptance of contingent fees and associated legal prohibitions, forensic practitioners strive to avoid providing professional services on the basis of contingent fees.”

The correct answer is: should ordinarily avoid providing his or her services on the basis of contingent fees.

How well did you know this?
1
Not at all
2
3
4
5
Perfectly
49
Q

You are asked by an attorney to evaluate the woman he is representing in a custody case and then recommend that she be granted custody in court. As an ethical psychologist, you should:
Select one:

a. evaluate the woman only if you can do a thorough assessment before making a recommendation on her behalf.
b. evaluate the woman only if you can do a thorough assessment and then make a recommendation on her behalf only if the results indicate that she is a good parent.
c. evaluate the woman and testify about the results but do not make specific recommendations about custody without evaluating the husband and children.
d. refuse to make a recommendation about which parent should be granted custody under any circumstances.

A

This issue is addressed in APA’s Guidelines for Child Custody Evaluations in Family Law Proceedings.

Answer C is correct: Any conclusions or recommendations about custody must be based on adequate information, which means that all parties should be evaluated before a recommendation is made. You could, however, evaluate the woman only and provide a summary of your results without making specific recommendations.

The correct answer is: evaluate the woman and testify about the results but do not make specific recommendations about custody without evaluating the husband and children.

How well did you know this?
1
Not at all
2
3
4
5
Perfectly
50
Q

You have been seeing 11-year-old Debbie in therapy for conduct problems at school for the past three months. Both of her parents are pleased with Debbie’s progress in therapy but are now getting a divorce and Debbie’s mother asks you to conduct a child custody evaluation of Debbie for the custody hearing. You should:
Select one:

a. refer Debbie to another psychologist for the evaluation.
b. request that the judge decide if you should perform the evaluation.
c. perform the evaluation if you are able to obtain consent from both parents.
d. perform the evaluation since you are the best person to do so since you have been working with Debbie.

A

This issue is addressed in APA’s Guidelines for Child Custody Evaluations in Family Law Proceedings, which is one of the APA documents that you want to be familiar with for the exam.

Answer A is correct: The Guidelines state that psychologists must avoid multiple relationships. For example, they should avoid an evaluative role in custody cases when they have already established a therapeutic relationship with the child and/or the child’s parent(s).

The correct answer is: refer Debbie to another psychologist for the evaluation.

How well did you know this?
1
Not at all
2
3
4
5
Perfectly
51
Q

You recently evaluated the defendant in a murder trial at the request of her attorney. The purpose of the evaluation was to support the defendant’s claim that she was insane at the time she committed the crime. The case received a great deal of public attention and you would now like to write a book about it and include information about the results of your evaluation. According to the Specialty Guidelines for Forensic Psychology:
Select one:

a. you may do so only with the consent of the defendant or the defendant’s legal representative.
b. you may do so as long as the information you include in your book is in the public record or, if not, as long as you have obtained the consent of the defendant.
c. you may do so only with the consent of the court.
d. you may not do so under any circumstances.

A

Because you conducted an evaluation of the defendant for the purpose of her defense, you may use information obtained from that evaluation for other purposes only under certain conditions.

Answer B is correct: This situation is addressed in Paragraphs 8.04 and 11.07 of the Specialty Guidelines. Paragraph 11.07 states: “Forensic practitioners strive to address particular legal proceedings in publications or communications only to the extent that the information relied upon is part of a public record, or when consent for that use has been properly obtained from any party holding any relevant privilege.”

The correct answer is: you may do so as long as the information you include in your book is in the public record or, if not, as long as you have obtained the consent of the defendant.

How well did you know this?
1
Not at all
2
3
4
5
Perfectly
52
Q

A psychologist has received a subpoena duces tecum requesting that she testify in court about a former client and provide the court with records related to the client’s treatment. The psychologist does not have a release from the client to do so. She should:
Select one:

a. appear in court and release the records as requested.
b. appear in court but claim the privilege on behalf of the client.
c. appear in court but release only those records she believes to be relevant to the case.
d. refuse to appear in court until she obtains a release from the client.

A

When a psychologist receives a subpoena to appear at a legal proceeding, he or she must appear but should not provide information about the client without a release from the client or a court order.

Answer B is correct: The appropriate action in this case is to appear as requested but, without a release from the client, to assert the privilege on the client’s behalf.

The correct answer is: appear in court but claim the privilege on behalf of the client.

How well did you know this?
1
Not at all
2
3
4
5
Perfectly
53
Q

A psychologist is subpoenaed to testify at a deposition about a current therapy client. This means that the psychologist:
Select one:

a. must testify about the client as requested.
b. must appear at the deposition as requested.
c. must provide the requested documents.
d. can expect to receive a court order within 60 days.

A

A subpoena is a legal document that requests the recipient to appear and testify at legal proceeding.

Answer B is correct: As noted by R. I. Simon (Clinical psychiatry and the law, Washington, DC, American Psychiatric Publishing, 2003), a subpoena by itself is not “proper legal compulsion” and only requires the psychologist to appear at a deposition or trial, not to testify. After receiving a subpoena, a psychologist should determine if the client has signed a release; if not, the psychologist should assert the privilege rather than provide the information requested in the subpoena when he/she appears at the deposition.

The correct answer is: must appear at the deposition as requested.

How well did you know this?
1
Not at all
2
3
4
5
Perfectly
54
Q

For a client to bring a claim of malpractice against his or her therapist, which of the following conditions is NOT necessary?
Select one:

a. The therapist must have had a professional relationship with the client that established a legal duty of care.
b. The harm or injury experienced by the client must be due to deliberate action or inaction on the part of the therapist.
c. There is a demonstrable standard of care that the therapist has breached.
d. The therapist’s breach of duty within the standard of care was the proximate cause of the harm or injury.

A

For the exam, you want to be familiar with the four conditions for a claim of malpractice. Malpractice is addressed in the chapter on Ethics and Professional Practice in the written study materials.

Answer B is correct: “Deliberate action or inaction on the part of the therapist” is not one of the four conditions for a claim of malpractice.

Answer A is incorrect: This is one of the conditions for a claim of malpractice.

Answer C is incorrect: This is one of the four conditions for a claim of malpractice.

Answer D is incorrect: This is another condition for a claim of malpractice.

The correct answer is: The harm or injury experienced by the client must be due to deliberate action or inaction on the part of the therapist.

How well did you know this?
1
Not at all
2
3
4
5
Perfectly
55
Q

Pope and Vetter (1992) found that psychotherapists encountered ethical dilemmas related to which of the following most frequently in their work?
Select one:

a. confidentiality
b. multiple relationships
c. competence
d. financial arrangements

A

Pope and Vetter (1992) surveyed over 690 APA members to obtain information on ethical dilemmas faced by these practitioners.

Answer A is correct: Results of the the survey found that confidentiality was the most frequently encountered “ethically troubling issue” by respondents.

The correct answer is: confidentiality

How well did you know this?
1
Not at all
2
3
4
5
Perfectly
56
Q

Research by Lamb and Catanzaro (1998) on sexual misconduct by psychotherapists found that:
Select one:

a. therapists who had sex with their clients often had sexual relations in the past with their own therapist, a professor, or supervisor.
b. therapists who had sex with their clients were more likely than those who did not to have been involved in nonsexual dual relationships with clients.
c. therapists who had sex with their clients were, in general, less experienced and younger than those who did not.
d. there were no consistent differences between therapists who did and did not have sex with their clients in terms of other dual relationships or sexual relations with their own therapist, professor, or supervisor.

A

The studies have found some differences in the characteristics of therapists who do and do not become sexually involved with clients. The results of this research are summarized in the Ethics and Professional Issues chapter of the written study materials.

Answer B is correct: Lamb and Catanzaro (1998) looked specifically at sexual and nonsexual boundary violations of psychotherapists and found that those reporting sexual boundary violations were also more likely to report nonsexual boundary violations and to rate nonsexual boundary violations as less negative or problematic than those who did not report sexual boundary violations.

The correct answer is: therapists who had sex with their clients were more likely than those who did not to have been involved in nonsexual dual relationships with clients.

How well did you know this?
1
Not at all
2
3
4
5
Perfectly
57
Q

The Examination for Professional Practice in Psychology (EPPP) is best described as:
Select one:

a. a way to protect the public from incompetent psychologists.
b. a measure of basic knowledge of psychology.
c. a predictor of job performance as a psychologist.
d. a way of determining if psychologists need additional training before being licensed.

A

The EPPP is a requirement for licensure in the United States and Canada.

Answer B is correct: As described by the ASPPB, the EPPP is designed “to evaluate the knowledge that should have been acquired by any candidate who is seeking licensure to practice psychology.” It has not been validated as a predictor of actual job performance or as a method for determining the need for additional training.

The correct answer is: a measure of basic knowledge of psychology.

How well did you know this?
1
Not at all
2
3
4
5
Perfectly
58
Q

The primary purpose of the state and provincial licensing boards is to:
Select one:

a. protect the profession of psychology.
b. protect the public.
c. define ethical standards of conduct.
d. define standards of care.

A

The primary purpose of the licensing boards is to protect the public.

Answer B is correct: To achieve this goal, the boards establish minimum standards of competence for psychologists including degree, supervision, and examination requirements.

The correct answer is: protect the public.

How well did you know this?
1
Not at all
2
3
4
5
Perfectly
59
Q

To compare the impact of two or more interventions on patients’ duration and quality of life, you would conduct which of the following types of cost analysis?
Select one:

a. cost-utility
b. cost-effectiveness
c. cost-minimization
d. cost-benefit

A

For the exam, you want to be familiar with the types of cost analysis described in the Ethics and Professional Issues chapter of the written study materials.

Answer A is correct: A cost-utility analysis is conducted to compare the effects of two or more treatments on duration and quality of life.

Answer B is incorrect: Cost-effectiveness analysis is used to compare the costs and benefits of different treatments in terms of specific nonmonetary outcomes (e.g., symptom severity, premature dropout rate) to identify the treatment that produces the best outcomes at the least cost.

Answer C is incorrect: As its name implies, a cost-minimization analysis is conducted to determine the least costly option among several options.

Answer D is incorrect: A cost-benefit analysis involves determining the relative costs and benefits of a particular treatment in monetary terms.

The correct answer is: cost-utility

How well did you know this?
1
Not at all
2
3
4
5
Perfectly
60
Q

To serve as an expert witness in a court case, you:
Select one:

a. must have the consent of the person you will be testifying about.
b. must testify only with regard to what you have personally observed.
c. must be qualified by the court to offer opinions and conclusions.
d. must be qualified by the attorneys for the plaintiff and defendant to offer opinions and conclusions.

A

An expert witness is allowed to draw conclusions from available data, even if the data were not personally observed or obtained.

Answer C is correct: Expert witnesses are qualified by the court to offer opinions and conclusions.

The correct answer is: must be qualified by the court to offer opinions and conclusions.

How well did you know this?
1
Not at all
2
3
4
5
Perfectly
61
Q

At the beginning of therapy with Mark M., you discussed the possible duration of treatment and estimated that therapy would not extend beyond his insurance coverage of your fee. You also informed Mark that you would be unable to reduce your fee when his therapy was no longer covered by insurance. Unfortunately, that time has arrived: Mark’s insurance will cover only the next three sessions, many of Mark’s problems are still unresolved, and he cannot afford your full fee. In terms of ethical responsibilities, you are:
Select one:

A. required to see Mark at a reduced or deferred fee until his problems are adequately resolved.
B. required to see Mark at a reduced or deferred fee until he feels ready to terminate treatment.
C. required to discuss the matter with Mark and make a referral at the end of the three sessions if necessary.
D. not required to continue seeing Mark since you discussed this matter at the beginning of therapy.

A

Standard 6.04 of the APA’s Ethics Code and Principle II.32 of the Canadian Code of Ethics apply to this situation.
a. Incorrect See explanation for response c.

b. Incorrect See explanation for response c.
c. CORRECT You are not required to continue seeing a client if the client cannot pay your fee. However, to avoid abandoning the client when insurance coverage has ended, you should either offer services at a reduced or deferred fee or, if that is not feasible, make appropriate referrals.
d. Incorrect Discussion of this issue at the beginning of therapy is a good practice but does not eliminate your responsibility to the client when his insurance runs out.

The correct answer is: required to discuss the matter with Mark and make a referral at the end of the three sessions if necessary.

How well did you know this?
1
Not at all
2
3
4
5
Perfectly
62
Q

After a year in private practice, you hire a newly-licensed psychologist whom you will provide with an office and secretarial services. You will also refer clients to the psychologist and pay him 50% of the fees he collects from those clients. This arrangement is:
Select one:

A. unethical because it involves barter.
B. unethical because it involves being paid for client referrals.
C. ethical as long as the psychologist is being compensated in an equitable way.
D. ethical because, as an employer, you can determine the conditions of employment.

A

The arrangement described in this question seems somewhat dubious in terms of acceptability, but the facts presented do not clearly indicate an ethical violation.

a. Incorrect See explanation for response c.
b. Incorrect See explanation for response c.
c. CORRECT Of the responses given, this one is the best. Referral fees are not entirely prohibited by ethical guidelines, which rules out responses a and b; and response d is not an accurate description of an employer’s ethical responsibilities. This issue is addressed in Standard 6.07 of the Ethics Code and is discussed in the Ethics Manual of the written study materials.
d. Incorrect See explanation for response c.

The correct answer is: ethical as long as the psychologist is being compensated in an equitable way.

How well did you know this?
1
Not at all
2
3
4
5
Perfectly
63
Q

An “informed consent” from a client:
Select one:

A. must always be in writing.
B. must always be in writing and in the client’s native language.
C. may be either verbal or in writing.
D. may be either verbal or in writing but must be documented.

A

The issue of informed consent is addressed in Standard 3.10 of the APA’s Ethics Code and Principle I.22 of the Canadian Code of Ethics.
a. Incorrect See explanation for response d.

b. Incorrect See explanation for response d.
c. Incorrect See explanation for response d.
d. CORRECT Standard 3.10 of the APA’s Ethic Code states that consents must be “appropriately documented,” and Principle I.22 of the Canadian Code of Ethics states that it is acceptable to “accept and document oral consents” when there is a good reason not to obtain a signed written consent. Therefore, this is the best answer of those given.

The correct answer is: may be either verbal or in writing but must be documented.

How well did you know this?
1
Not at all
2
3
4
5
Perfectly
64
Q

A client whom Dr. Oliver Olivetti has been seeing for several months has recently changed jobs, and the client has learned that her new insurance plan does not begin covering pre-existing conditions for 12 months. The client asks Dr. Olivetti if he could write his bills so that it appears that she has just started therapy. If Dr. Olivetti complies with the client’s request, he is acting:
Select one:

A. ethically since he is taking the client’s welfare into consideration.
B. ethically only if not to do so would require the client to quit therapy.
C. ethically only if the insurance forms do not require him to sign a statement saying that treatment began after the effective date of the insurance.
D. unethically.

A

The client in this situation is asking Dr. Olivetti to commit insurance fraud and, if Dr. Olivetti does so, he will be acting both unethically and illegally.

a. Incorrect Although Dr. Olivetti should consider his client’s welfare (by, for example, offering a reduced fee so that the client doesn’t have to terminate therapy), he would be acting unethically and illegally if he agrees to the arrangement requested by the client.
b. Incorrect This arrangement would be illegal and unethical regardless of the ramifications for the client.
c. Incorrect This would still represent insurance fraud since the client and therapist are well aware of the insurance provisions.
d. CORRECT As noted above, complying with the client’s request would represent insurance fraud, which would be both unethical and illegal. See, e.g., Standard 6.04 of the APA’s Ethics Code.

The correct answer is: unethically.

How well did you know this?
1
Not at all
2
3
4
5
Perfectly
65
Q

Which of the following best describes the status of professional (therapy) records?
Select one:

A. The physical record belongs to the client.
B. The physical record belongs to the therapist.
C. The physical record belongs to the therapist, but the information contained therein belongs to the client.
D. The physical record and the information contained therein belong jointly to the therapist and the client.

A

The laws regarding professional records are complex and vary somewhat from jurisdiction to jurisdiction. Keep in mind that, when answering questions like this one, you’ll want to pick a general answer that is likely to apply to most (or all) jurisdictions.
a. Incorrect See explanation for response c.

b. Incorrect See explanation for response c.
c. CORRECT While the actual physical record is the property of the therapist, laws require therapists to make the information contained in them available to the client in most situations. This does not mean that a therapist must release the physical record to a client but, instead, that the therapist may be required to show the client the record or provide the client with a summary or copy of it.
d. Incorrect See explanation for response c.

The correct answer is: The physical record belongs to the therapist, but the information contained therein belongs to the client.

How well did you know this?
1
Not at all
2
3
4
5
Perfectly
66
Q

Dr. Bettina Browne, a licensed psychologist, undertakes a child custody evaluation at the request of the child’s mother. The father agrees to be evaluated also, and Dr. Browne obtains signed informed consents from both parents before beginning her assessment. Dr. Browne administers the MMPI and Rorschach tests to both parents. The mother obtains significantly elevated scores on the MMPI’s K and L scales, suggesting that she tends to lie and to be excessively defensive, and these tendencies are confirmed by her Rorschach responses. No significant abnormality is suggested by the father’s MMPI and Rorschach results. On the basis of these test results, Dr. Browne recommends that the mother not be awarded custody of the child. Dr. Browne:
Select one:

A. has acted ethically because her evaluation included both parents.
B. has acted ethically because the parents signed informed consents before the evaluation.
C. has acted unethically because of her use of the MMPI and Rorschach tests as methods of evaluating the mothers parental suitability.
D. has acted unethically because she made specific recommendations regarding custody rather than simply reporting the results of the evaluation.

A

A psychologist is ethically required to use tests only for purposes for which they have been validated. See, e.g., Standard 9.02 of the APA’s Ethics Code.

a. Incorrect Although the psychologist acted appropriately by evaluating both parents prior to making a recommendation, response c is a better answer.
b. Incorrect Obtaining signed informed consents does not overcome the problem of using the MMPI and Rorschach as predictors of parental functioning, a purpose for which they have not been validated.
c. CORRECT Based on the information given, it appears that Dr. Browne has acted unethically by basing her custody recommendation on MMPI and Rorschach test results since these tests have not been validated for this purpose. Also, even if the tests had been validated for this purpose, Dr. Browne’s recommendation should have been based on more than the results of only two tests.
d. Incorrect Recommendations are acceptable as long as they are based on a thorough evaluation of all parties.

The correct answer is: has acted unethically because of her use of the MMPI and Rorschach tests as methods of evaluating the mothers parental suitability.

How well did you know this?
1
Not at all
2
3
4
5
Perfectly
67
Q

Miranda M., age 32, makes an appointment with you at the recommendation of her physician who has diagnosed her recurrent headaches as the result of stress. The woman tells you that a friend of hers was trained in self-hypnosis for tension headaches and says that she also wants training in self-hypnosis. You took a weekend workshop on hypnosis in graduate school but have not used it in your clinical practice. As an ethical psychologist, you should:
Select one:

A. admit your lack of expertise in the use of self-hypnosis and refer the woman to another therapist.
B. advise the woman that the underlying cause of her headaches would probably not be dealt with by self-hypnosis and suggest that she begin individual psychotherapy.
C. accept the woman for treatment and begin training her in self-hypnosis after reviewing your workshop notes.
D. accept the woman for treatment and begin training her in self-hypnosis but obtain consultation if you encounter any problems during the course of treatment.

A

Providing services within the boundaries of one’s competence is addressed by Standard 2.01 of the APA’s Ethic Code and Principle II.6 of the Canadian Code of Ethics.

a. CORRECT Of the responses given, this one is most consistent with the requirement to provide services that are within the boundaries of your professional competence. Note that, while it is up to you to determine your competence with regard to a particular service or technique, a weekend workshop in self-hypnosis is clearly inadequate.
b. Incorrect It would be unethical to try to convince the client to choose another course of treatment simply because you are not trained in the treatment she desires.
c. Incorrect Reviewing your workshop notes would not be adequate training.
d. Incorrect Although this response comes closer to fulfilling the requirements of the ethical guidelines than do responses b and c, it falls short of those requirements because it implies that consultation would not be sought unless you believe that you are having problems during the course of treatment. Consequently, response a is a better answer.

The correct answer is: admit your lack of expertise in the use of self-hypnosis and refer the woman to another therapist.

How well did you know this?
1
Not at all
2
3
4
5
Perfectly
68
Q

You have been hired to assist with a research project on process variables in group psychotherapy. The psychologist who is the principal investigator tells you to discourage participants from leaving the group prior to the end of the study because, if too many participants drop out, the study’s potential usefulness will be seriously compromised. As an ethical psychologist, you should:
Select one:

A. refuse to assist with the research project.
B. immediately file a complaint against the psychologist with the psychology licensing board or ethics committee.
C. write a letter to the psychologist, indicating the relevant ethical standards and offering to discuss the matter with him.
D. follow the psychologist’s instructions since, by “discouraging” participants from leaving the group, you are not actually coercing them to participate.

A

Standard 8.02 of the APA’s Ethics Code and Principle I.30 of the Canadian Code of Ethics apply to this situation. Both require psychologists to allow participants to withdraw from a research study at any time.

a. Incorrect Resigning from the project would not correct the ethical violation.
b. Incorrect Although a report may eventually be necessary, it is not the best first step since this matter may be amenable to an informal resolution (answer c).
c. CORRECT By offering to discuss the matter with the psychologist, you are attempting to resolve the matter informally, which is consistent with the requirements of the ethical guidelines.
d. Incorrect This would be unethical, and in so doing, you would be violating the provisions of Standards 1.04 and 1.05 of the Ethics Code and Principles II.40 and II.41 of the Canadian Code of Ethics, which require psychologists to take action when a colleague acts in an unethical way.

The correct answer is: write a letter to the psychologist, indicating the relevant ethical standards and offering to discuss the matter with him.

How well did you know this?
1
Not at all
2
3
4
5
Perfectly
69
Q

A psychologist is the supervisor of several interns at a community hospital in a small town and notices that one of the interns seems unenthusiastic and depressed. When he talks to her, she tells him that she’s been very unhappy lately and feels she has no one to turn to. The intern asks the psychologist to see her in therapy, and he agrees to do so. The psychologist has acted:
Select one:

A. unethically by agreeing to get involved in a dual relationship.
B. unethically by violating the intern’s right to privacy.
C. ethically because they are working in a community hospital in a small town.
D. ethically because the intern is in desperate need of help.

A

In most situations, psychologists should avoid multiple (dual) relationships. See, e.g., Standard 3.05(a) of APA’s Ethics Code and Principle III.33 of the Canadian Code of Ethics.

a. CORRECT Standard 3.05(a) of the Ethics Code states that “a psychologist refrains from entering into a multiple relationship if the multiple relationship could reasonably be expected to impair the psychologist’s objectivity, competence, or effectiveness in performing his or her functions as a psychologist, or otherwise risks exploitation or harm to the person with whom the professional relationship exists.” The psychologist in the situation described in this question is entering into a multiple relationship that may threaten his objectivity and effectiveness as a supervisor and therapist.
b. Incorrect There is no indication that privacy is an issue in this situation.
c. Incorrect In some circumstances, multiple relationships may be unavoidable. However, the fact that the intern and psychologist work in a community hospital in a small town does not necessarily mean that no other options are available.
d. Incorrect There is no indication that the intern is in such “desperate need of help” that she cannot wait to see another professional.

The correct answer is: unethically by agreeing to get involved in a dual relationship.

How well did you know this?
1
Not at all
2
3
4
5
Perfectly
70
Q

A psychologist decides to use a computerized test service to facilitate scoring and interpretation of the MMPI-2, which he frequently administers to his clients. The psychologist should be aware that:
Select one:

A. the use of computerized interpretations is prohibited by ethical guidelines.
B. computerized scoring and interpretation services are notoriously unreliable.
C. computerized interpretations should always be supplemented with other information obtained by the psychologist.
D. because of their objectivity, computerized interpretations are preferable to subjective interpretations.

A

Automated scoring and interpretation services are available for a number of psychological tests including the MMPI-2.

a. Incorrect The use of automated interpretation services is not prohibited by ethical guidelines as long as certain standards are met by both the service and the users of the service.
b. Incorrect The quality of computer interpretations varies from service to service, and it cannot be concluded that all computerized interpretations are “notoriously unreliable.”
c. CORRECT The information in a computerized interpretation is limited and, therefore, the best policy is to always supplement computerized interpretations with information from other tests, interviews, observations, etc. See, e.g., Standards 9.02 and 9.09 of the APA’s Ethics Code.
d. Incorrect The information provided by computerized interpretations is limited and the accuracy of the interpretation depends on the quality of the service. Therefore, it cannot be concluded that computerized interpretations are necessarily the preferred method of score interpretation.

The correct answer is: computerized interpretations should always be supplemented with other information obtained by the psychologist.

How well did you know this?
1
Not at all
2
3
4
5
Perfectly
71
Q

The term “insanity” is a:
Select one:

A. popular (lay) term for a mental disorder.
B. legal, nonpsychiatric term for certain types of mental disorders.
C. psychiatric term that refers to a psychotic disorder with an unknown etiology.
D. psychiatric term that refers to an organic mental disorder.

A

Insanity is a legal term that varies somewhat in terms of its definition from jurisdiction to jurisdiction.
a. Incorrect See explanation for response b.

b. CORRECT The term “insanity” is used in the legal arena and is not a psychiatric term. Generally, a defendant who pleads not guilty by reason of insanity is contending that he was precluded by a mental disease or defect from understanding the difference between right and wrong.
c. Incorrect See explanation for response b.
d. Incorrect See explanation for response b.

The correct answer is: legal, nonpsychiatric term for certain types of mental disorders.

How well did you know this?
1
Not at all
2
3
4
5
Perfectly
72
Q

Alice A., who has a master’s degree in clinical psychology but is not licensed, is hired by a community mental health clinic to, among other things, administer several standard psychological tests to adult clients. According to ethical guidelines, this is:
Select one:

A. acceptable as long as a professional clinical psychologist co-signs Alice’s evaluations.
B. acceptable as long as a professional clinical psychologist supervises Alice’s activities.
C. unacceptable since Alice is not a licensed psychologist.
D. unacceptable since Alice does not have a doctoral degree.

A

Ethical (and legal) guidelines do not prohibit the provision of all psychological services by individuals who are not licensed clinical psychologists. However, they do require that such persons be supervised by a professional clinical psychologist.

a. Incorrect This would not be considered adequate supervision.
b. CORRECT This answer is most consistent with ethical and legal guidelines. Alice may administer some psychological tests as long as she has been adequately trained and is supervised by a professional clinical psychologist. See, e.g., Standard 9.07 of he APA’s Ethics Code.
c. Incorrect See explanation above.
d. Incorrect See explanation above.

The correct answer is: acceptable as long as a professional clinical psychologist supervises Alice’s activities.

How well did you know this?
1
Not at all
2
3
4
5
Perfectly
73
Q

Following the retirement of his business partner, Dr. Sheldon Smythe is the only psychologist in a small town, and the next closest mental health professional works in a town that is over 100 miles away. Several of Dr. Smythe’s new clients have problems for which he has limited training and experience. Dr. Smythe should:
Select one:

A. refuse to see the clients.
B. see these clients but use only interventions that he has experience using.
C. see the clients and obtain supervision or consultation by telephone.
D. agree to see these clients only if they are experiencing a crisis.

A

Although psychologists should ordinarily provide services that are within the boundaries of their training and experience, there are exceptions.

a. Incorrect See explanation for response c.
b. Incorrect See explanation for response c.
c. CORRECT It is acceptable and desirable for psychologists to acquire new skills and to treat new problems - but only when appropriate supervision or consultation is obtained. The situation described in this question is a complex one, but this response is the best one given the circumstances (the next closest mental professional works in a town that is over 100 miles away) and the alternatives provided in the other answers.
d. Incorrect See explanation for response c.

The correct answer is: see the clients and obtain supervision or consultation by telephone.

74
Q

A 24-year-old woman calls Dr. Sylvia Silby for an appointment but refuses to discuss her problem on the phone. During the first therapy session, Dr. Silby finds the client to be an attractive woman, dressed in a somewhat flashy manner. The young woman discusses her difficulties in a vague, guarded manner, indicating that, for the past few months, she has had increasing difficulty sleeping and has felt anxious and irritable. Ten minutes before the end of the session, the young woman tells Dr. Silby that she has been seeing another therapist for six months and that, about four weeks ago, she started having a sexual relationship with him. As an ethical psychologist, Dr. Silby should:
Select one:

A. advise the woman to terminate treatment with the other psychologist and begin therapy with her.
B. explain to her the seriousness of her allegations and advise her of her rights and alternative courses of action.
C. contact the other therapist to discuss the matter with him.
D. file a complaint against the other therapist with the ethics committee and recommend to the woman that she immediately terminate treatment with him.

A

If the woman’s allegations are true, the original therapist has violated ethical guidelines and may have violated the law. Ethical violations by colleagues are covered in Standards 1.04 and 1.05 of the APA’s Ethics Code and are discussed in the Ethics Manual of the written study materials.

a. Incorrect See explanation for response b.
b. CORRECT Dr. Silby cannot contact the therapist or make a report herself (answers c and d) without the client’s consent since to do so would represent a breach of confidentiality. Therefore, of the actions listed in the answers, the best one is to discuss the matter with the client and advise her of her options.
c. Incorrect See explanation for response b.
d. Incorrect See explanation for response b.

The correct answer is: explain to her the seriousness of her allegations and advise her of her rights and alternative courses of action.

75
Q

Faaris F. terminated therapy with Dr. Pamela Prince several months ago but calls to request an appointment because he is now very lonely and depressed and feels “like he wants to die.” In the past two months, Dr. Prince has been reducing her caseload because she is planning to retire. The best course of action in this situation is for Dr. Prince to:
Select one:

A. explain to Faaris that she is reducing her caseload but that she is willing to counsel him by phone in case of an emergency.
B. see Faaris in therapy until the crisis has passed and then provide him with appropriate referrals.
C. explain to Faaris that she is no longer accepting clients and refer him to a colleague.
D. provide Faaris with several referrals and ask him to call her if none of those turn out to be satisfactory.

A

The situation described in this question is not explicitly addressed by ethical guidelines, so the best strategy is to choose the answer that best addresses the client’s welfare.

a. Incorrect See explanation for response b.
b. CORRECT Since the client seems to be experiencing a crisis (he feels “like he wants to die”), this course of action takes his welfare into account and is most consistent with the “spirit” of the ethical guidelines. See, for example, Standards 3.04 and 10.09 of the APA’s Ethics Code and Principles II.2 and II.34 of the Canadian Code of Ethics.
c. Incorrect See explanation for response b.
d. Incorrect See explanation for response b.

The correct answer is: see Faaris in therapy until the crisis has passed and then provide him with appropriate referrals.

76
Q

Dr. Wanda Weim, a licensed psychologist, receives a request to testify in a custody proceeding on behalf of a man that she had been seeing in conjoint marital therapy until six months ago. The man wants Dr. Weim to provide the court with information about his parenting skills and his relationship with his wife and children. Dr. Weim contacts the man’s wife, who refuses to give the psychologist permission to testify. Dr. Weim should:
Select one:

A. testify as requested by the husband.
B. testify only about matters pertaining to the husband.
C. testify only about matters for which she is qualified to make a judgment.
D. refuse to testify.

A

Multi-client situations create special considerations and concerns about confidentiality since it is not often possible to release information about one client without also revealing confidential information about another client.

a. Incorrect See explanation for response d.
b. Incorrect See explanation for response d.
c. Incorrect See explanation for response d.
d. CORRECT In this situation, both clients (the husband and wife) must give their permission before Dr. Weim can release confidential information. Therefore, he should refuse to testify (unless, of course, he is ordered to do so by the court).

The correct answer is: refuse to testify.

77
Q

You have just started seeing a client who is from a different cultural background than you are from who is having difficulty coping with the recent death of her mother. You have frequently worked with clients who are coping with loss, but you have never worked with a client from this background. You’d like to continue seeing this client in therapy, however, because you want to broaden your experience to include clients from diverse cultural backgrounds. Your best course of action would be to:
Select one:

A. continue working with the client but proceed with caution.
B. seek supervision or consultation and continue to work with the client.
C. refer the client to another therapist who has experience working with clients from this background.
D. explain the situation to the client and terminate therapy.

A

It is acceptable (and desirable) for psychologists to acquire new skills and to treat new problems - but only when appropriate supervision or consultation is obtained.

a. Incorrect See explanation for response b.
b. CORRECT This situation is addressed by Standard 2.01(b) of the APA’s Ethics Code and Principle II.8 of the Canadian Code of Ethics. Standard 2.01(b) states “where scientific or professional knowledge in the discipline of psychology establishes that an understanding of factors associated with age, gender, gender identity, race, ethnicity, culture, national origin, religion, sexual orientation, disability, language, or socioeconomic status is essential for effective implementation of their services or research, psychologists have or obtain the training, experience, consultation, or supervision necessary to ensure the competence of their services, or they make appropriate referrals.” This answer is consistent with this requirement and makes the most sense, given the fact that there is no indication that a referral is necessary (i.e., the question indicates that you have previously worked with clients who have experienced a loss).
c. Incorrect See explanation for response b.
d. Incorrect See explanation for response b.

The correct answer is: seek supervision or consultation and continue to work with the client.

78
Q

After three sessions with a therapy client, Dr. Leonard Lykowski realizes that he’s feeling somewhat hostile toward the client because she reminds him of his wife who he’s currently divorcing. Dr. Lykowski’s best course of action would be to:
Select one:

A. refer the client to another therapist without telling the client the reason for the referral.
B. refer the client to another therapist after discussing the reason with her.
C. recognize that his feelings are the result of countertransference and continue to work with the client.
D. consult with another psychologist to determine whether or not to continue seeing the client in therapy.

A

In the situation described in this question, Dr. Lykowski’s personal problems may be affecting his objectivity.

a. Incorrect See explanation for response d.
b. Incorrect See explanation for response d.
c. Incorrect Although Dr. Lykowski’s feelings represent countertransference, he would want to seek consultation to determine if those feelings are negatively impacting (or are likely to negatively impact) his ability to provide the client with appropriate services.
d. CORRECT Consultation with another professional is ordinarily the best course of action whenever a therapist is concerned that a personal problem might interfere with his/her ability to provide a client with adequate therapy. This issue is addressed in Standard 2.06 of the APA’s Ethics Code and is discussed in the Ethics Manual of the written study materials.

The correct answer is: consult with another psychologist to determine whether or not to continue seeing the client in therapy.

79
Q

Dr. Mansfield Mayhem receives a subpoena requiring him to testify about a current therapy client at a trial. Dr. Mayhem contacts the client who states that she does not want him to release any confidential information to the court. Dr. Mayhem should:
Select one:

A. notify the attorney who issued the subpoena that the client has asserted the privilege and request that he be released from subpoena.
B. do nothing further until he receives an order from the court to appear at the trial.
C. notify the court that he will not be appearing at the trial because the client does not want him to do so.
D. advise the attorney who issued the subpoena that he cannot appear as requested because the client has not given him permission to do so.

A

A subpoena is a legal document that requires a psychologist to appear at a designated time at a legal proceeding. If the client invokes the privilege (says she does not want the therapist to release confidential information to the court), the subpoena is still valid and the psychologist must appear unless he/she has been released from it.

a. CORRECT When the client invokes the privilege in this type of situation, the psychologist should contact the client’s attorney or his/her own attorney to discuss the matter. The psychologist can also contact the attorney who issued the subpoena to request to be released from it. Unless the psychologist receives a written release, he/she must appear as requested but would not reveal confidential (privileged) information without the client’s consent or without an order from the court to do so.
b. Incorrect See explanation for response a.
c. Incorrect See explanation for response a.
d. Incorrect See explanation for response a.

The correct answer is: notify the attorney who issued the subpoena that the client has asserted the privilege and request that he be released from subpoena.

80
Q

You receive an e-mail from Dr. Brenda Browne, a licensed psychologist, who is currently seeing one of your former clients for a substance use disorder. Dr. Browne asks you to forward the client’s file to her and states that the client has signed a release authorizing you to do so. You should:
Select one:

A. wait until you receive a request from the client before taking any action.
B. contact the client to discuss the release of information.
C. forward a photocopy of the file to the psychologist as requested.
D. forward a summary of the file to the psychologist that includes only information related to the clients current condition.

A

Answer B is correct. Even though the person requesting the client’s record is a licensed psychologist, you would not want to breach confidentiality by providing confidential client information to Dr. Browne without knowing specifically what the client has consented to. The psychologist would want to verify that the client has, in fact, signed a release and discuss the release and its potential consequences with the client before providing Dr. Browne with the requested information.

The correct answer is: contact the client to discuss the release of information.

81
Q

“Privilege”:
Select one:

A. is an ethical requirement designed to protect client information from being disclosed except when the client has signed a release.
B. is a legal requirement designed to protect confidential information from being disclosed to anyone not directly involved in a client’s case.
C. is an ethical requirement that applies to the confidentiality of therapist-client communications in both legal and nonlegal situations.
D. is a legal requirement designed to protect confidential information from being disclosed in a legal proceeding.

A

Privilege is a legal term and refers to “privileged communications” that arise in legally designated relationships (e.g., the therapist-client relationship).
a. Incorrect This answer does not accurately describe privilege.

b. Incorrect This answer does not accurately describe privilege.
c. Incorrect Privilege is a legal requirement to maintain confidentiality in legal situations.
d. CORRECT Privilege is “a legal term describing certain specific types of relationships that enjoy protection from disclosure in legal proceedings” [G. P., Koocher, Privacy, confidentiality, and privilege, in G. P. Koocher, J. C. Norcross & S. S. Hill (eds.), Psychologists’ Desk Reference, Oxford University Press, New York, 1998]. Privilege is discussed in the Ethics Manual of the written study materials.

The correct answer is: is a legal requirement designed to protect confidential information from being disclosed in a legal proceeding.

82
Q

A woman comes to family therapy with her three daughters, ages 10, 13, and 19, all of whom live at home with the woman and their father. During the third therapy session, the 19-year-old tells you that her father began sexually molesting her when she was 13. When you ask the younger girls if their father has ever abused them, they are reluctant to say anything about their father, and the mother adamantly denies any knowledge of abuse. In this situation, you should:
Select one:

A. advise the mother to contact the appropriate authorities.
B. file a report with the appropriate authorities yourself.
C. meet with the father to determine if he is abusing the younger daughters.
D. maintain confidentiality since the known victim of abuse is over 18.

A

Whenever you acquire a reasonable suspicion in your professional capacity that a minor is being abused, you have a legal obligation to report the abuse.

a. Incorrect See explanation for response b.
b. CORRECT Although psychologists are not usually required to make a report when the individual is over the age of 18, in the situation described in the question, you would have reasonable suspicion that the younger sisters are also victims of abuse. Therefore, you would be legally required to file a report with the appropriate authorities. Additional information about reporting child abuse is provided in the Ethics Manual of the written study mateirals.
c. Incorrect See explanation for response b.
d. Incorrect See explanation for response b.

The correct answer is: file a report with the appropriate authorities yourself.

83
Q

You receive a letter from Mrs. Bisque’s attorney asking you to testify that Mrs. Bisque, who is in the process of a divorce settlement, should have sole custody of her children. You are aware that Mr. Bisque was recently admitted to a psychiatric hospital in another city. You should:
Select one:

A. make a recommendation based on your knowledge that Mr. Bisque is obviously unable to have custody of the children.
B. make a recommendation based on an interview with Mrs. Bisque and her children.
C. refuse to make a recommendation since psychologists are not supposed to make specific custody recommendations.
D. refuse to make a specific recommendation about custody unless you can evaluate the entire family.

A

Although it is not unethical to provide information (e.g., test results) about a single party in a custody dispute, it would be unethical to make a recommendation about custody based on incomplete information.

a. Incorrect See explanation for response d.
b. Incorrect See explanation for response d.
c. Incorrect See explanation for response d.
d. CORRECT An accepted standard of practice in custody situations is that a psychologist should not make a recommendation for custody unless the psychologist has evaluated all parties. This is consistent with the requirements of Standard 9.01 of APA’s Ethics Code and Principle II.13 of the Canadian Code of Ethics and with the provisions of the APA’s Guidelines for Child Custody Evaluations in Divorce Proceedings.

The correct answer is: refuse to make a specific recommendation about custody unless you can evaluate the entire family.

84
Q

You receive a request from a psychologist working at a mental health crisis unit for information about a former client of yours who left three months ago with an unpaid bill of over $500. The client has been admitted as an inpatient and has signed a release. You tell the psychologist that you will provide her with the information she has requested only after the client has made arrangements for paying you the money he owes. You have:
Select one:

A. acted ethically as long as you had informed the client of your policy regarding unpaid fees.
B. acted ethically since the client has violated your implicit contract with him to pay the fee for the services you provided to him.
C. acted ethically since this action is not covered by the ethical guidelines.
D. acted unethically.

A

In this situation, the former client’s records are needed for an emergency situation (i.e., the client has been admitted as an inpatient to a mental health crisis unit).

a. Incorrect See explanation for response d.
b. Incorrect See explanation for response d.
c. Incorrect See explanation for response d.
d. CORRECT The ethical guidelines prohibit “holding a client’s records hostage” because the client has not paid his or her bill when these records are requested and needed for emergency treatment. See Standard 6.03 of the APA’s Ethics Code which explicitly prohibits this practice and Principles II.1 and 2 of the Canadian Code of Ethics which require psychologists to promote the welfare of clients and to avoid harming them.

The correct answer is: acted unethically.

85
Q

Kameko K., a psychology intern, administers psychological tests to the clients of a community mental health clinic, but her supervisor always scores the tests and interprets their results. When preparing the psychological report for these clients, the supervisor does not indicate that the intern administered the tests. This is:
Select one:

A. ethical because the supervisor scored and interpreted the tests.
B. ethical because the supervisor is responsible for the accuracy of the psychological report.
C. unethical because an intern should not be administering psychological tests.
D. unethical because the intern should be listed as the examiner in the report.

A

This issue is not directly addressed by ethical guidelines, so the best approach when choosing the answer would have been to identify the one that is most consistent with the “spirit” of the guidelines.

a. Incorrect See explanation for response d.
b. Incorrect See explanation for response d.
c. Incorrect See explanation for response d.
d. CORRECT This response is most consistent with ethical guidelines. For example, Standard 5.01(b) of the APA’s Ethics Code prohibits psychologists from making misleading statements and Standard 6.06 requires psychologists to provide accurate information to payors and funding sources. It is also consistent with Principles III.1 and 5 of the Canadian Code of Ethics.

The correct answer is: unethical because the intern should be listed as the examiner in the report.

86
Q

A client that Dr. William Wong has been seeing in therapy for six months is currently the defendant in a court case. The client’s attorney contacts Dr. Wong, requesting that he provide her with information about the client’s diagnosis and treatment progress since this information will be helpful in preparing the client’s defense. Dr. Wong should:
Select one:

A. release the information as requested since privilege is waived in this situation.
B. release only the information he believes is relevant to the clients case.
C. release the information since the request came from the clients attorney.
D. discuss the release with the client before taking any additional action.

A

The release of confidential client information ordinarily requires a waiver of confidentiality from the client.

a. Incorrect See explanation for response d.
b. Incorrect See explanation for response d.
c. Incorrect See explanation for response d.
d. CORRECT Privilege may or may not be waived in this situation and, even if it is, Dr. Wong should discuss the release of confidential information with the client before doing so (and, in most cases, obtain a waiver from the client).

The correct answer is: discuss the release with the client before taking any additional action.

87
Q

During a discussion with a 4th grade teacher, the teacher tells a school psychologist that one of her students is constantly disrupting the class. After evaluating the child and obtaining more information about the situation, the psychologist concludes that the teacher’s approach with the child is part of the problem. Consequently, the psychologist should:
Select one:

A. make an appointment with the principal to discuss the teacher’s approach.
B. explain to the teacher how her approach may be exacerbating the problem.
C. recommend that he (the therapist) start counseling the child on a regular basis.
D. recommend that the child be transferred to another classroom

A

Although this situation is not explicitly addressed by ethical guidelines, the APA’s Ethics Code and the Canadian Code of Ethics require psychologists to cooperate with other professionals.

a. Incorrect See explanation for response b.
b. CORRECT General Principle B of the APA’s Ethics Code states that “psychologists consult with, refer to, or cooperate with other professionals and institutions to the extent needed to serve the best interests of those with whom they work.” School psychologists must often act as a coordinator to bring together a variety of people and resources to resolve problems. Of the answers given, this one is most in line with the child’s best interests and the school psychologist’s responsibility to work with all people involved in the case.
c. Incorrect See explanation for response b.
d. Incorrect See explanation for response b.

The correct answer is: explain to the teacher how her approach may be exacerbating the problem.

88
Q

To serve as an expert witness in a court case, a psychologist must:
Select one:

A. be board certified as an expert witness.
B. be qualified by the court to offer opinions and conclusions about the case.
C. have been hired by the person he/she will be testifying about.
D. testify only about what he/she personally observed or obtained.

A

An expert witness is a person who, because of his/her special training, knowledge, or experience, is qualified to offer an opinion as testimony in a court proceeding.

a. Incorrect See explanation for response b.
b. CORRECT Expert witnesses are qualified by the court to offer opinions and conclusions.
c. Incorrect See explanation for response b.
d. Incorrect See explanation for response b.

The correct answer is: be qualified by the court to offer opinions and conclusions about the case.

89
Q

Sally S., age 14, tells her therapist that she wants to die and that she has access to her mother’s sleeping pills. The therapist believes that Sally’s threat is serious and tells Sally that he’ll have to contact her parents. Sally says she thought they had agreed when she started therapy that everything she told the therapist would be confidential. If the therapist contacts Sally’s parents, he has acted:
Select one:

A. unethically if Sally was, in fact, told that confidentiality would be maintained.
B. unethically if he contacts Sallys parents without first obtaining Sallys consent to do so.
C. ethically as long as the limits of confidentiality were discussed with Sally at the outset of therapy.
D. ethically since a breach of confidentiality is justified when a client is suicidal.

A

Psychologists are legally and ethically obligated to breach confidentiality whenever a client is believed to be a danger to him/herself.
a. Incorrect See explanation for response d.

b. Incorrect See explanation for response d.
c. Incorrect See explanation for response d.
d. CORRECT As noted above, psychologists must take appropriate action whenever a client is suicidal. Because Sally is a minor, contacting her parents would be an appropriate action. Exceptions to confidentiality and privilege are addressed in the Ethics Manual of the written study materials.

The correct answer is: ethically since a breach of confidentiality is justified when a client is suicidal.

90
Q

You receive a call from Mrs. Wang who is very upset because her 14-year-old daughter witnessed the assault of her best friend three days ago and is very distraught. The girl hasn’t slept, won’t eat, and can’t stop crying. You have limited experience working with adolescents and in providing crisis intervention services. However, there is no one else in the community who is more experienced than you are. As an ethical psychologist, you will:
Select one:

A. agree to see Mrs. Wang’s daughter in therapy since you’ve had some experience providing crisis intervention experiences.
B. inform Mrs. Wang about your lack of experience and let her decide if she wants you to provide therapy to her daughter.
C. inform Mrs. Wang that you cannot see her daughter because of your lack of experience.
D. see Mrs. Wang’s daughter in therapy only until the crisis has ended or until you locate alternative services.

A

Standard 2.02 of the APA’s Ethics Code and Principle II.8 of the Canadian Code of Ethics apply to this situation.
a. Incorrect See explanation for response d.

b. Incorrect See explanation for response d.
c. Incorrect See explanation for response d.
d. CORRECT This answer is most consistent with ethical requirements. Standard 2.02 of the Ethics Code, for example, states that “in emergencies, when psychologists provide services to individuals for whom other mental health services are not available and for which psychologists have not obtained the necessary training, psychologists may provide such services in order to ensure that services are not denied. The services are discontinued as soon as the emergency has ended or appropriate services are available.”

The correct answer is: see Mrs. Wang’s daughter in therapy only until the crisis has ended or until you locate alternative services.

91
Q

Which of the following is true about the use of deception in a research study that may cause participants physical pain?
Select one:

A. Deception is acceptable in studies that may cause pain only when the use of pain is justified by the study’s potential value.
B. Deception is acceptable in studies that may cause pain only when there are no alternative methods for conducting the study.
C. Deception is prohibited when the pain is physical but not when it is psychological in nature.
D. Deception is prohibited whether the pain is physical or psychological in nature.

A

Answer D is correct. Deception in research is acceptable only when certain conditions are met. This answer is most consistent with Standard 8.07(b) of the APA’s Ethics Code and Principle III.24 of the Canadian Code of Ethics. Standard 8.07(b), for example, states that “psychologists do not deceive prospective participants about research that is reasonably expected to cause physical pain or severe emotional distress.”

The correct answer is: Deception is prohibited whether the pain is physical or psychological in nature.

92
Q

Dr. Calvin Claymore, a licensed psychologist, regularly waives the co-payment for low-income clients whose therapy fees are covered by insurance. This practice is:
Select one:

A. ethical only if the insurance company has agreed to this arrangement.
B. ethical since it is in the best interests of low-income clients.
C. ethical as long as Dr. Claymore doesnt bill the insurance company for an increased hourly fee in order to collect her full fee.
D. ethical as long as the insurance company does not explicitly prohibit this arrangement.

A

The practice described in this question represents insurance fraud, which is both unethical and illegal.

a. CORRECT Most insurance companies agree to pay a specific percentage of a psychologist’s fee but, when the co-payment is waived, this means the insurance company is paying the entire fee. Consequently, regularly waiving the co-payment without the permission of the insurance company represents insurance fraud.
b. Incorrect See explanation for response a.
c. Incorrect See explanation for response a.
d. Incorrect See explanation for response a.

The correct answer is: ethical only if the insurance company has agreed to this arrangement.

93
Q

A psychologist is hired by a court to evaluate a defendant during the course of a criminal proceeding. The psychologist can subsequently include information about the defendant in a textbook that she is writing on forensic psychology:
Select one:

A. as long as the defendant was warned of the limits of confidentiality at the time of the evaluation.
B. only if the psychologist has received the permission of the defendants legal representative to do so.
C. as long as the psychologist determines that the value of the information warrants its inclusion in the book and she disguises all personally identifiable information.
D. as long as the information included in the book is included in the public record of the case and she disguises all personally identifiable information.

A

This issue is addressed in Paragraph 8.04 of the APA’s Specialty Guidelines for Forensic Psychology.

a. Incorrect This would be insufficient.
b. Incorrect This is true if the information to be included in the book is not part of the public record.
c. Incorrect This is untrue.
d. CORRECT This answer is most consistent with the provisions of Paragraph 8.04, which states: “Forensic practitioners using case materials for purposes of teaching, training, or research strive to present such information in a fair, balanced, and respectful manner. They attempt to protect the privacy of persons by disguising the confidential, personally identifiable information of all persons and entities who would reasonably claim a privacy interest; using only those aspects of the case available in the public domain; or obtaining consent from the relevant clients, parties, participants, and organizations to use the materials for such purposes.”

The correct answer is: as long as the information included in the book is included in the public record of the case and she disguises all personally identifiable information.

94
Q

A psychologist receives a phone call from a person whom she had an intimate (including sexual) relationship with for a short period of time over two years ago. The relationship ended amicably, and this person wants to start therapy with the psychologist. With regard to ethical requirements, which of the following is true about this situation?
Select one:

A. It would be ethical to begin therapy with this person as long as the psychologist does not revive their previous personal relationship.
B. It would be ethical to begin therapy with this person since two years have passed since the end of the relationship.
C. It would be ethical to begin therapy with this person as long as the psychologist discusses the potential for conflicts during the informed consent process.
D. It would not be ethical to begin therapy with this person.

A

This situation is directly addressed in Standard 10.07 of the APA’s Ethics Code and indirectly addressed by Principle III.33 of the Canadian Code of Ethics.
a. Incorrect See explanation for response d.

b. Incorrect See explanation for response d.
c. Incorrect See explanation for response d.
d. CORRECT This answer is most consistent with Standard 10.07, which explicitly prohibits psychologists to accept former sexual partners as therapy clients and Principle III.33, which prohibits multiple relationships that might impair a psychologist’s objectivity.

The correct answer is: It would not be ethical to begin therapy with this person.

95
Q

When conducting research with human participants, a psychologist should be aware that:
Select one:

A. participants must be debriefed as soon as possible after their participation in the study, especially when the study has involved deception.
B. participants must be debriefed after their participation in the study only if the psychologist believes that not doing so will result in harm to them.
C. participants must be debriefed after their participation only if the study places them at high risk.
D. debriefing of research participants is at the discretion of the psychologist.

A

Debriefing of research participants is addressed in Standard 8.08 of the APA’s Ethic Code and Principles II.44 and III.26 of the Canadian Code of Ethics.

a. CORRECT Standard 8.08 of the Ethics Code requires psychologists to inform participants promptly following their participation in the study about its nature, results, and conclusions; and Principle III.26 of the Canadian Code of Ethics requires psychologists to debrief participants as soon as possible after their involvement in a study when there has been incomplete disclosure about the study’s purpose. Consequently, of the answers given, this one is most consistent with ethical guidelines.
b. Incorrect See explanation for response a.
c. Incorrect See explanation for response a.
d. Incorrect See explanation for response a.

The correct answer is: participants must be debriefed as soon as possible after their participation in the study, especially when the study has involved deception.

96
Q

A psychologist should be aware that information contained in client records:
Select one:

A. is protected by law and can never be disclosed without the client’s consent.
B. is the property of the psychologist and cannot be disclosed without his/her consent.
C. is the joint property of the psychologist and client and cannot be disclosed without the consent of both.
D. is not free from disclosure all of the time, regardless of the wishes of the client or the psychologist.

A

There are some situations when client information must be released, despite the wishes of the client or the psychologist.

a. Incorrect See explanation for response d.
b. Incorrect See explanation for response d.
c. Incorrect See explanation for response d.
d. CORRECT Record ownership and access are governed by legal guidelines. In most jurisdictions, the laws grant ownership of the physical record to the practitioner but give clients the right to have access to the information contained in the record. In addition, there are times when other individuals or entities have the right to access client information (e.g., confidentiality of client information is waived in certain situations in the military and privilege is waived in certain legally defined situations). Therefore, this is the best answer of those given.

The correct answer is: is not free from disclosure all of the time, regardless of the wishes of the client or the psychologist.

97
Q

A psychologist in private practice is going through a difficult divorce and, as a result, has started drinking and is often depressed. He should:
Select one:

A. plunge deeper into his work to take his mind off his own problems.
B. seek professional consultation and consider temporarily suspending his practice.
C. explain the situation to his clients and offer to refer them to other therapists.
D. immediately refer his clients to other therapists.

A

This issue is addressed by Standard 2.06(b) of the APA’s Ethics Code and Principle II.11 of the Canadian Code of Ethics.

a. Incorrect See explanation for response b.
b. CORRECT Psychologists do not necessarily have to abandon their professional practice when experiencing a period of personal distress. However, when personal problems may be adversely affecting the welfare of their clients, psychologists must take appropriate action. This often includes “obtaining professional consultation or assistance … to determine whether they should limit, suspend, or terminate their work-related duties” [Ethics Code, Standard 2.06(b)].
c. Incorrect See explanation for response b.
d. Incorrect This may be necessary, but the best initial step would be to seek consultation to determine the most appropriate course of action.

The correct answer is: seek professional consultation and consider temporarily suspending his practice.

98
Q

A psychology intern offers his supervisor a chance to invest in a real estate syndicate that is being arranged by the intern’s father as a tax shelter. The supervisor:
Select one:

A. would be foolish to refuse.
B. should invest but only in his wife’s name.
C. should refuse to invest.
D. should report the intern to the ethics committee.

A

The situation described in this question represents a multiple (dual) relationship that could potentially impact the psychologist’s ability to remain objective when acting as a supervisor for the intern.

a. Incorrect See explanation for response c.
b. Incorrect See explanation for response c.
c. CORRECT Multiple relationships are addressed in Standard 3.05 of the APA’s Ethics Code and Principle III.33 of the Canadian Code of Ethics. Standard 3.05 states that “a psychologist refrains from entering into a multiple relationship if the multiple relationship could reasonably be expected to impair the psychologist’s objectivity, competence, or effectiveness in performing his or her functions as a psychologist, or otherwise risks exploitation or harm to the person with whom the professional relationship exists.” Principle III.33 of the Canadian Code of Ethics includes a similar requirement.
d. Incorrect It would be a better strategy for the supervisor to educate the intern about multiple relationships than to report the intern to the ethics committee.

The correct answer is: should refuse to invest.

99
Q

Dr. Molly Mendez, a well-known behavioral psychologist, is asked by her secretary if she would be willing to provide him with therapy to help him overcome a gambling addiction. If Dr. Mendez agrees to do so, she will be acting:
Select one:

A. ethically as long as she has expertise in treating gambling addiction.
B. ethically as long as she charges the secretary her usual fee for psychological services.
C. unethically since doing so is likely to create an unacceptable multiple relationship.
D. unethically since maintaining the secretary’s confidentiality will be difficult in this situation.

A

The situation described in this question represents a multiple relationship - i.e., Dr. Mendez would be acting as both employer and therapist if she agrees to see her secretary in therapy.
a. Incorrect See explanation for response c.

b. Incorrect See explanation for response c.
c. CORRECT A potentially harmful multiple relationship is the primary problem in the situation described in this question. Multiple relationships are addressed in Standard 3.05 of the APA’s Ethics Code and Principle III.33 of the Canadian Code of Ethics. These guidelines prohibit multiple relationships that could be expected to impair a psychologist’s objectivity, which is a potential problem in this situation since both roles (employer and therapist) would continue simultaneously over an extended period of time.
d. Incorrect See explanation for response c.

The correct answer is: unethically since doing so is likely to create an unacceptable multiple relationship.

100
Q

Dr. Julian Jones is given a research idea by a colleague during the course of a casual conversation and decides to design and conduct a study to follow-up on the idea. When publishing the results of the study, it would be best for Dr. Jones to:
Select one:

A. offer his colleague joint authorship as first author.
B. offer his colleague joint authorship as second author.
C. acknowledge his colleague’s contribution in a footnote or introductory statement.
D. publish the study without acknowledging his colleague since the colleagues contribution was minor.

A

This issue is addressed by Standard 8.12 of the APA’s Ethics Code and Principle III.7 of the Canadian Code of Ethics.

a. Incorrect See explanation for response c.
b. Incorrect See explanation for response c.
c. CORRECT Standard 8.12(b) states that publication credit must “accurately reflect the relative scientific or professional contributions of the individuals involved, regardless of their relative status”; and Principle III.7 states that psychologists “give credit for work done or ideas contributed by others … in proportion to their contribution.” While the idea for the study came from the colleague, she did not participate in designing or carrying out the study and it would be acceptable to acknowledge her contribution in an introductory statement or footnote.
d. Incorrect See explanation for response c.

The correct answer is: acknowledge his colleague’s contribution in a footnote or introductory statement.

101
Q

A psychologist is writing a book for popular distribution and illustrates the section on projective personality tests with a number of the pictures used in the Hand Test. This use of test materials in this way is:
Select one:

A. clearly unethical because it compromises the usefulness of the test.
B. ethical only if the publisher of the book has accepted responsibility for including the pictures.
C. ethical since the Hand Test is not a commonly used psychological test.
D. professionally questionable but not covered by ethical guidelines.

A

Answer A is correct: This answer is most consistent with the provisions of Standard 9.11 of the APA’s Ethics Code and Principle IV.11 of the Canadian Code of Ethics. Standard 9.11 states that “psychologists make reasonable efforts to maintain the integrity and security of test materials and other assessment techniques”; and Principle IV.11 states that psychologists “protect the skills, knowledge, and interpretations of psychology from being misused, used incompetently, or made useless (e.g., loss of security of assessment techniques) by others.”

b. Incorrect See explanation for response a.
c. Incorrect See explanation for response a.
d. Incorrect See explanation for response a.

The correct answer is: clearly unethical because it compromises the usefulness of the test.

102
Q

During the course of treatment, a women reports to her therapist, Dr. Emil Emerson, that her husband hit somebody, causing the victim a mild concussion. If Dr. Emerson maintains the client’s confidentiality, he has acted:
Select one:

A. ethically since confidentiality in this situation should not be breached.
B. ethically since the woman’s report is only hearsay.
C. unethically since he should have reported the incident to the appropriate authorities.
D. unethically since he should have contacted the intended victim.

A

The situation described in this question is not one that mandates a breach of confidentiality.

a. CORRECT A breach of client confidentiality is acceptable and, in some cases, mandated by law. However, this situation involves a past incident involving the client’s husband, and a breach of confidentiality is not acceptable or required. See, for example, Standard 4.05(b) of the APA’s Ethics Code, which states that exceptions to client confidentiality are permitted only when a breach is necessary to avoid violating the law, or when breaching confidentiality would “protect the client/patient, psychologist, or others from harm.”
b. Incorrect See explanation for response a.
c. Incorrect See explanation for response a.
d. Incorrect See explanation for response a.

The correct answer is: ethically since confidentiality in this situation should not be breached.

103
Q

Dr. Beatrice Bowen, a licensed psychologist, has been contacted by a company to evaluate job applicants. The job will involve administering projective tests to determine if applicants for an accountant job have a “homosexual orientation.” Dr. Bowen should:
Select one:

A. not accept the job because the evaluation she is being requested to conduct represents an invasion of privacy.
B. not accept the job because projective tests have been shown to be unreliable and invalid.
C. accept the job and administer the tests as required but explain their purpose to all applicants before doing so.
D. accept the job, administer the tests as required, and report the results to the company.

A

A “homosexual orientation” is not relevant to the ability to perform an accountant job successfully and to seek such information as part of a job-related assessment is clearly an invasion of privacy.

a. CORRECT This answer is most consistent with ethical guidelines. Standard 9.04(b) of the APA’s Ethics Code, for example, states that “psychologists may refrain from releasing test data to protect a client/patient or others from substantial harm or misuse or misrepresentation of the data or the test”; and Principle I.38 of the Canadian Code of Ethics states that psychologists “take care not to infringe … on the personally, developmentally, or culturally defined private space of individuals or groups, unless clear permission is granted to do so.”
b. Incorrect See explanation for response a.
c. Incorrect See explanation for response a.
d. Incorrect See explanation for response a.

The correct answer is: not accept the job because the evaluation she is being requested to conduct represents an invasion of privacy.

104
Q

A client is in therapy with you as part of the requirements for her probation. During the third therapy session, she tells you that she wants to quit therapy. You should:
Select one:

A. tell her that she cannot do so.
B. tell her that she can quit but that she needs to consider the legal ramifications of doing so.
C. tell her that her desire to quit is normal resistance and that she should come to therapy for at least two more sessions before making a decision.
D. call the police immediately.

A

The situation described in this question is addressed by Standard 3.07 of the APA’s Ethics Code and Principle I.26 of the Canadian Code of Ethics.
a. Incorrect See explanation for response b.

b. CORRECT This answer is most consistent with Standard 3.07 and Principle I.26, which address a psychologist’s responsibilities when providing services at the request of a third party. In this situation, the psychologist must ensure that the client understands what legal sanctions she may face if she does not stay in therapy.
c. Incorrect See explanation for response b.
d. Incorrect See explanation for response b.

The correct answer is: tell her that she can quit but that she needs to consider the legal ramifications of doing so.

105
Q

A ___________ is a person appointed by the court to represent and make decisions in a legal proceeding for a minor or other person legally incapable of doing so.
Select one:

A. testamentary guardian
B. guardian ad litem
C. conservator of a minor
D. special conservator

A

For the licensing exam, you’ll want to be familiar with the term that is being asked about by this question. Additional information about guardianship is provided in the Ethics Manual of the written study materials.

a. Incorrect A testamentary guardian is a person who is appointed in a deed or will to serve as guardian of a minor or disabled person.
b. CORRECT The description presented in this question best describes a guardian ad litem who is appointed by the court for a specific purpose (e.g., to make decisions for someone who is incapable of doing so).
c. Incorrect A conservator of a minor is responsible for managing the minor’s finances.
d. Incorrect A special conservator carries out the terms of a preliminary protective order.

The correct answer is: guardian ad litem

106
Q

Dr. Delores Del Rio has been seeing Jeff J. in therapy for four months. For the last month, Jeff has complained about the progress he is making in therapy and, Dr. Del Rio has responded by working with him to modify treatment goals. In the last therapy session, however, Jeff again stated that he is not happy with therapy. Dr. Del Rio’s best course of action would be to:
Select one:

A. recognize that Jeff’s concerns are a normal reaction to the changes he is making as the result of therapy.
B. make a clinical decision as to whether or not Jeff is actually benefiting from therapy and, if he is, encourage him to continue with therapy for at least several more sessions.
C. discuss Jeff’s continued dissatisfaction with him and the possibility of terminating therapy.
D. tell Jeff that she is ethically required to terminate therapy with him since he is not satisfied with the progress he is making.

A

This question addresses the issue of terminating therapy when a client does not seem to be benefiting from it.

a. Incorrect See explanation for response c.
b. Incorrect See explanation for response c.
c. CORRECT This is the best answer of those given because it takes the client’s welfare into consideration and is most consistent with ethical guidelines regarding termination of professional services [see Standard 10.10(a) of the APA’s Ethics Code or Principle II.37 of the Canadian Code of Ethics].
d. Incorrect See explanation for response c.

The correct answer is: discuss Jeff’s continued dissatisfaction with him and the possibility of terminating therapy.

107
Q

Dr. Donald Dreadmire is starting a therapy group for recently divorced men. He tells the editor of the local newspaper (who has just divorced his wife) that he can attend the group for free if the editor includes an article about the program in the newspaper. Dr. Dreadmire has acted:
Select one:

A. ethically since his request does not violate ethical guidelines.
B. ethically as long as the editor accurately describes the program.
C. unethically because his request is exploitative and violates ethical guidelines.
D. unethically only if the arrangement negatively impacts his objectivity when working with the editor in therapy.

A

This issue is directly addressed by Standard 5.02(b) of the APA’s Ethics Code and indirectly addressed by Principle III.31 of the Canadian Code of Ethics.

a. Incorrect See explanation for response c.
b. Incorrect See explanation for response c.
c. CORRECT Of the answers given, this one is most consistent with ethical guidelines. Standard 5.02(b) of the Ethics Code prohibits psychologists from compensating “employees of press, radio, television, or other communication media in return for publicity in a news item”; and Principle III.31 of the Canadian Code of Ethics prohibits psychologists from exploiting “any relationship established as a psychologist to further personal, political, or business interests at the expense of the best interests of their clients.”
d. Incorrect See explanation for response c.

The correct answer is: unethically because his request is exploitative and violates ethical guidelines.

108
Q

You are hired as an adjunct professor to teach a course at a graduate school of professional psychology. In terms of ethical requirements, you are required to ensure that:
Select one:

A. accurate information about the course content is available for all interested parties.
B. the course content addresses the most recent research and theory in the area.
C. the course content addresses the interests of the students who enroll in the class.
D. students are given an opportunity to evaluate you at the end of the term.

A

This issue is addressed by Standard 7.02 of the APA’s Ethics Code and Principle III.5 of the Canadian Code of Ethics.

a. CORRECT Ethical guidelines explicitly require psychologists to provide interested parties (in this case, the school administration and the students) with accurate and complete information about course content.
b. Incorrect Although presenting the most recent research and theory is usually a good idea, the importance of doing so would depend on the nature of the class. For example, recent research and theory would not be covered in a “history of psychology” class.
c. Incorrect The interests of the students do not necessarily coincide with the content that you, as the instructor, believe is important to present.
d. Incorrect Providing students with opportunities to evaluate the course is desirable but is not required by ethical guidelines.

The correct answer is: accurate information about the course content is available for all interested parties.

109
Q

In general, which of the following is considered the “best defense” for a psychologist who is being sued for malpractice?
Select one:

A. adequate records
B. a signed informed consent
C. providing state-of-the-art services
D. good malpractice insurance

A

Most experts agree that good record-keeping is the best defense for practitioners in cases of litigation.

a. CORRECT In addition to being familiar with relevant laws and ethical guidelines, the best way to reduce vulnerability to malpractice is maintaining adequate records. See, e.g., S. Knapp, Professional liability and risk management in an era of managed care, in D. T. Marsh and R. D. Magee (eds.), Ethical and legal issues in professional practice with families (pp. 271-288), New York, John Wiley & Sons, 1997.
b. Incorrect See explanation for response a.
c. Incorrect See explanation for response a.
d. Incorrect See explanation for response a.

The correct answer is: adequate records

110
Q

A 26-year-old client you have been seeing for two months unexpectedly admits to you that he had sexual relations with the 15-year old daughter of his neighbor six months ago. He says that he knows what he did was wrong and that he’ll never do it again. You should:
Select one:

A. maintain the client’s confidentiality.
B. report the client to the appropriate authorities.
C. alert the parents of his victim.
D. report the client to the appropriate authorities and alert the parents of his victim.

A

Answer B is correct. In cases of known and suspected child abuse, a psychologist is legally and ethically required to make a report to the appropriate authorities. You would want to contact the appropriate authorities in this situation, but contacting the parents of the victim (answers c and d) would represent a breach of the client’s confidentiality.

The correct answer is: report the client to the appropriate authorities.

111
Q

Dr. Sam Stone has been hired by a consulting firm to assist with an ongoing research project being conducted at a large company. Dr. Stone is to work directly under the project director, Dr. Thompson, a licensed organizational psychologist. Dr. Thompson asks Dr. Stone to review the preliminary report she has prepared for the project; and, in doing so, Dr. Stone discovers that some of the reported data is misleading and may be inaccurate. Dr. Stone expresses his concerns to Dr. Thompson and is told that “it will be taken care of.” Subsequently, Dr. Stone learns that the report was submitted to the company without any changes. Dr. Stone reviews the ethical guidelines and decides that pursuing the matter further isn’t necessary. Dr. Stone’s conclusion:
Select one:

A. is correct since this situation is not covered by ethical guidelines.
B. is correct since he has already attempted to resolve the matter in an informal manner as proscribed by ethical guidelines.
C. is correct since it is the project directors responsibility to provide the company with complete and accurate data.
D. is incorrect because he has a responsibility to ensure that the company is provided with complete and accurate data.

A

This issue is addressed by Standard 1.01 of the APA’s Ethics Code and Principle II.5 of the Canadian Code of Ethics.

a. Incorrect See explanation for response d.
b. Incorrect See explanation for response d.
c. Incorrect See explanation for response d.
d. CORRECT This answer is most consistent with the “spirit” of ethical guidelines and, more specifically with Standard 1.01 of the Ethics Code, which states that “if psychologists learn of misuse or misinterpretation of their work, they take reasonable steps to correct or minimize the misuse or misrepresentation”; and with Principle II.5 of the Canadian Code of Ethics, which states that psychologists “make every reasonable effort to ensure that psychological knowledge is not misused, intentionally or unintentionally, to harm others.” (Also see Standard 5.01 of the Ethics Code and Principle III.1 of the Canadian Code of Ethics.)

The correct answer is: is incorrect because he has a responsibility to ensure that the company is provided with complete and accurate data.

112
Q

The mother of a 31-year-old client that you have been seeing for six months calls to check on her son’s progress. She says that she is very concerned about her son and wants to know if there is anything she can do to help him. As an ethical psychologist, you:
Select one:

A. advise her in general terms about how she can help her son.
B. suggest that she make an appointment so you can discuss the matter in person.
C. suggest that she come to therapy with her son at his next appointment.
D. tell her that you cannot discuss her son’s condition with her.

A

This question addresses the issue of client confidentiality.

a. Incorrect See explanation for response d.
b. Incorrect See explanation for response d.
c. Incorrect See explanation for response d.
d. CORRECT Giving the mother any information about her adult son would represent a breach of his confidentiality. Therefore, this is the best answer of those given.

The correct answer is: tell her that you cannot discuss her son’s condition with her.

113
Q

Dr. Karen Kempfer, a licensed psychologist who specializes in couple and marriage therapy, has tested positive for HIV. In terms of ethical requirements, Dr. Kempfer:
Select one:

A. should seek supervision to ensure that her medical condition does not negatively impact her ability to provide effective services to therapy clients.
B. should refrain from participating in any professional activities that might be adversely affected by her medical condition.
C. should inform her therapy clients of her medical condition so that they can decide whether they want to continue seeing her.
D. is not obligated to take any precautions because of her medical condition.

A

Standard 2.06 of the APA’s Ethics Code and Principle II.11 of the Canadian Code of Ethics apply to the situation described in this question.

a. Incorrect Although Dr. Kempfer should take appropriate action if she believes her condition may interfere with her effectiveness as a therapist, supervision may not be the best course of action.
b. CORRECT This answer is most consistent with ethical guidelines that address the impact of a psychologist’s social, emotional, and health-related problems on his/her professional services. Standard 2.06 requires psychologists to “refrain from initiating an activity when they know or should know there is a substantial likelihood that their personal problems will prevent them from performing their work-related activities in a competent manner”; and Principle II.11 similarly requires psychologists to obtain appropriate help and/or discontinue professional activities when “a physical or psychological condition reduces their ability to benefit and not harm others.”
c. Incorrect It is not necessary for Dr. Kempfer to inform her clients of her condition.
d. Incorrect Dr. Kempfer would want to take some action if her medical condition is impacting her ability to provide effective psychological services.

The correct answer is: should refrain from participating in any professional activities that might be adversely affected by her medical condition.

114
Q

To be consistent with ethical requirements, David DeFoe, a psychology intern, must do which of the following when a supervisor has legal responsibility for his work?
Select one:

A. He must tell his clients that he is a psychology intern.
B. He must tell his clients that his work is being supervised.
C. He must provide clients with information about his education and experience.
D. He must indicate his status on his business card.

A

Standard 10.01 of the APA’s Ethics Code and Principle III.22 of the Canadian Code of Ethics apply to the situation described in this question.

a. CORRECT This answer is most consistent with ethical guidelines that require interns (trainees) to inform their clients of their professional status. Note that Standard 10.01(c) of the Ethics Code also requires that Mr. DeFoe provide clients with the name of his supervisor.
b. Incorrect Telling clients that he is being supervised would not be adequate since this would not clearly indicate that he is an intern.
c. Incorrect See explanation for response a.
d. Incorrect See explanation for response a.

The correct answer is: He must tell his clients that he is a psychology intern.

115
Q

A client Dr. Amanda Alto has been seeing in therapy for three months is suddenly fired from her job and can no longer afford Dr. Alto’s fee. The client suggests that she babysit for Dr. Alto’s children in exchange for therapy. In terms of ethical guidelines, this exchange would be:
Select one:

A. ethical.
B. ethical only if Dr. Alto discusses the potential for conflict with the client.
C. ethical only if a fair market value is established for the clients babysitting services.
D. unethical.

A

The situation described in this question is addressed by ethical guidelines related to multiple relationships and therapy fees.

a. Incorrect See explanation for response d.
b. Incorrect See explanation for response d.
c. Incorrect See explanation for response d.
d. CORRECT The situation described in this question violates ethical guidelines regarding multiple relationships and therapy fees. With regard to multiple relationships, if Dr. Alto agrees to the arrangement, she will be acting as both therapist and employer, which may impair her ability to provide effective therapy to the client. In addition, the arrangement represents a form of barter (which is explicitly addressed by Standard 6.05 of the APA’s Ethics Code and indirectly addressed by Principles II.1 and 2 of the Canadian Code of Ethics) that could also interfere with her objectivity.

The correct answer is: unethical.

116
Q

Dr. Nancy Noonan plans to conduct a study that will involve testing and interviewing psychiatric inpatients. Before beginning the study, she must obtain:
Select one:

A. informed consent from the hospitals administrator.
B. informed consents from the patients legal guardians.
C. informed consents from the patients legal guardians and assent from the patients.
D. assent from the patients.

A

Standards 3.10 and 8.02 of the APA’s Ethics Code and Principle I.35 of the Canadian Code of Ethics apply to the situation described in this question.

a. Incorrect See explanation for response c.
b. Incorrect See explanation for response c.
c. CORRECT The answers given to this question imply that the patients are unable to give informed consent to their participation in the study since this option is not provided in any of the answers. Therefore, this is the best answer of those given. In most cases, whenever a person is unable to provide an informed consent for participation in research (i.e., because he/she is incapable of doing so), a psychologist must obtain consent from the person’s legal guardian and assent (agreement) from the person.
d. Incorrect See explanation for response c.

The correct answer is: informed consents from the patients legal guardians and assent from the patients.

117
Q

A psychology intern who has been working at a mental health clinic for the past six months tells you that his internship was suddenly terminated by his supervisor for “incompetence.” He also says that he had not been given any warning that his performance was inadequate before being terminated. You tell the intern that:
Select one:

A. the supervisor’s behavior seems unfair but is not addressed by ethical guidelines.
B. the supervisor’s behavior is acceptable as long as the intern was given specific reasons for his termination.
C. the supervisor’s behavior is acceptable as long as the reason for the interns termination is valid.
D. the supervisor’s behavior may have violated ethical guidelines regarding supervisee performance evaluations.

A

Although termination of supervisees is not directly addressed by ethical guidelines, evaluating and providing feedback to supervisees is addressed by Standard 7.06 of the APA’s Ethics Code and Principle II.25 of the Canadian Code of Ethics.

a. Incorrect See explanation for response d.
b. Incorrect See explanation for response d.
c. Incorrect See explanation for response d.
d. CORRECT This answer is most consistent with ethical guidelines regarding evaluation and feedback as well as with more general provisions regarding the avoidance of harm (e.g., Standard 3.04 of the Ethics Code and Principles I.13 and II.2 of the Canadian Code of Ethics).

The correct answer is: the supervisor’s behavior may have violated ethical guidelines regarding supervisee performance evaluations.

118
Q

Which of the following best describes ethical guidelines regarding barter as a substitute for the collection of fees for professional psychological services?
Select one:

A. Barter may be acceptable when certain conditions are met.
B. The barter of goods (but not services) is prohibited by ethical guidelines.
C. The barter of services (but not goods) is prohibited by ethical guidelines.
D. Barter of any kind is prohibited by ethical guidelines.

A

Barter is explicitly addressed by Standard 6.05 of the APA’s Ethics Code and indirectly addressed by Principles III.31-33 of the Canadian Code of Ethics.

a. CORRECT Barter is prohibited by Standard 6.05 of the APA’s Ethics Code when it is clinically contradindicated or exploitative - in other words, in certain circumstances, barter may be acceptable. The Canadian Code of Ethics does not directly address barter but, to be consistent with the “spirit” of the Code, barter should be avoided when it may be harmful to the client.
b. Incorrect See explanation for response a.
c. Incorrect See explanation for response a.
d. Incorrect See explanation for response a.

The correct answer is: Barter may be acceptable when certain conditions are met.

119
Q

Which of the following is not a condition for a claim of malpractice against a psychologist?
Select one:

A. The psychologist must have a legal duty to the client.
B. There must be evidence that the psychologists actions were not in the best interests of the client.
C. The psychologist must have acted in a negligent or otherwise improper manner.
D. There must be a causal relationship between the psychologists negligence and the harm claimed by the client.

A

A claim of malpractice requires that four conditions be met: a duty to the client; a breach of that duty; harm to the patient; and a causal relationship between the breach of duty and harm to the patient.

a. Incorrect This is one of the four conditions for a malpractice claim.
b. CORRECT This does not accurately describe one of the four conditions.
c. Incorrect This also describes a condition for a claim of malpractice.
d. Incorrect This is one of the four conditions of malpractice.

The correct answer is: There must be evidence that the psychologists actions were not in the best interests of the client.

120
Q

Dr. Kay Klein, a licensed psychologist, is preparing a brochure for her weekend workshop on “Effective Parenting Strategies for Single Parents.” Included in the brochure are several testimonials from former workshop participants. According to ethical guidelines, this use of testimonials is:
Select one:

A. clearly ethical.
B. ethical if certain conditions are met.
C. unethical only when the testimonials are false or misleading.
D. clearly unethical.

A

The use of testimonials is addressed by Standard 5.05 of the APA’s Ethics Code and Principle III.31 of the Canadian Code of Ethics.

a. Incorrect See explanation for response b.
b. CORRECT Standard 5.05 states that psychologists do not solicit testimonials “from current therapy clients/patients or other persons who because of their particular circumstances are vulnerable to undue influence”; and Principle III.31 prohibits psychologists from exploiting clients. Therefore, of the answers given, this one is most consistent with ethical guidelines.
c. Incorrect Although false or misleading testimonials would be unethical, this answer is not the best one because it states that testimonials are unethical only when they contain false or misleading information.
d. Incorrect See explanation for response b.

The correct answer is: ethical if certain conditions are met.

121
Q

Malpractice is a tort involving which of the following?
Select one:

A. failure to protect
B. coercion
C. negligence
D. breach of contract

A

A tort is a civil wrongdoing resulting from a breach of duty.

a. Incorrect Although a tort involves a breach of duty, it is not necessarily related to the duty to protect (although a failure to protect in certain legally defined situations might result in a claim of malpractice).
b. Incorrect See explanation for response c.
c. CORRECT Malpractice is a civil tort involving a negligent breach of duty that results in harm to the client. Being familiar with the conditions for a claim of malpractice would have helped you identify this as the correct answer (see the Ethics Manual in the written study materials).
d. Incorrect See explanation for response c.

The correct answer is: negligence

122
Q

Of the following, which is responsible for the largest proportion of complaints filed with the APA’s Ethics Committee?
Select one:

A. sexual misconduct
B. test misuse
C. breach of confidentiality
D. discrimination

A

Although the types of actions that underlie complaints filed with the APA’s Ethics Committee vary in number from year to year, one of the actions listed in the answers to this question has consistently been the most frequent cause of complaints.

a. CORRECT The 2009 Report of the Ethics Committee, for example, identifies sexual misconduct of an adult as being the behavior most commonly cited in cases opened by the Ethics Committee, which is consistent with previous reports. Test misuse, breach of confidentiality, and discrimination (answers b, c, and d) are causes of complaints but not as often as is sexual misconduct.
b. Incorrect See explanation for response a.
c. Incorrect See explanation for response a.
d. Incorrect See explanation for response a.

The correct answer is: sexual misconduct

123
Q

You are contacted by a defendant’s attorney to evaluate the defendant to determine if there are grounds for the insanity defense. Who is the “holder of the privilege” in this situation?
Select one:

A. the defendant
B. the defendant’s attorney
C. the court
D. you (the psychologist)

A

Keep in mind that privilege is waived only in certain circumstances and that, even when it is waived, another person or agency does not become the “holder of the privilege.”

a. CORRECT The defendant would still be the “holder of the privilege” even if privilege is waived. (Note that, in the situation described in this question, privilege would be waived only if the defendant actually claims insanity as his/her defense during the trial.) Additional information on privilege is provided in the Ethics Manual in the written study materials.
b. Incorrect See explanation for response a.
c. Incorrect See explanation for response a.
d. Incorrect See explanation for response a.

The correct answer is: the defendant

124
Q

Which of the following is most likely a violation of ethical guidelines?
Select one:

A. a brochure containing client testimonials mailed to potential attendees at a conference for adult children of alcoholics
B. an advertisement offering a free initial therapy session that is published in the campus newspaper just prior to final exams
C. uninvited in-person solicitations for therapy of individuals whose homes were destroyed several months ago by a hurricane
D. a display ad for psychotherapy services in the yellow pages of the local telephone directory

A

Of the advertisements/solicitations described in the responses, only one is clearly prohibited by ethical guidelines.

a. Incorrect Testimonials are acceptable as long as they are not solicited from clients who are susceptible to undue influence (see Standard 5.05 of the APA’s Ethics Code).
b. Incorrect Free initial sessions are not prohibited by ethical guidelines.
c. CORRECT In-person solicitation of individuals who are vulnerable to undue influence is explicitly prohibited by Standard 5.06 of the APA’s Ethics Code and is inconsistent with the “spirit” of the Canadian Code of Ethics (see, e.g., Principles I.27 and III.31).
d. Incorrect Ads in the yellow pages of the phone directory are not prohibited by ethical guidelines.

The correct answer is: uninvited in-person solicitations for therapy of individuals whose homes were destroyed several months ago by a hurricane

125
Q

Dr. Harry Holliday, a clinical psychologist, is planning to take a three-week cruise. In terms of his clinical practice, Dr. Holliday’s best course of action would be to:
Select one:

A. discuss his vacation plans with his current clients ahead of time so that they know he’ll be unavailable during that time.
B. give his clients a phone number where he can be reached.
C. give his clients the phone number of a colleague who has agreed to accept phone calls in emergencies.
D. hire a colleague to take over his appointments while he is on vacation.

A

To be consistent with ethical requirements (e.g., Standard 3.12 of the APA’s Ethics Code), Dr. Holliday should make some kind of arrangements for his clients while he is on vacation.

a. Incorrect See explanation for response c.
b. Incorrect See explanation for response c.
c. CORRECT Of the answers given, this one makes the most sense in terms of the client’s welfare and Dr. Holliday’s interests. Ethically, Dr. Holliday would not be required to be available by phone to his clients during his vacation or to provide alternative ongoing services but would be required to provide the clients with a contact for emergency situations.
d. Incorrect See explanation for response c.

The correct answer is: give his clients the phone number of a colleague who has agreed to accept phone calls in emergencies.

126
Q

Which of the following best describes the provisions of the APA’s Specialty Guidelines for Forensic Psychology regarding contingent fees?
Select one:

A. Contingent fees are acceptable as long as they are fair and equitable.
B. Contingent fees are acceptable only for expert or fact witnesses who do not have a previous relationship with the person they are testifying about.
C. Contingent fees should ordinarily be avoided.
D. Contingent fees are not addressed by the Specialty Guidelines.

A

Contingent fees are fees that depend on the outcome of a case (often a percentage of the money the client receives).

a. Incorrect See explanation for response c.
b. Incorrect See explanation for response c.
c. CORRECT Contingent fees are addressed in Paragraph 5.02 of the Specialty Guidelines, which states: “Forensic practitioners seek to avoid undue influence that might result from financial compensation or other gains. Because of the threat to impartiality presented by the acceptance of contingent fees and associated legal prohibitions, forensic practitioners strive to avoid providing professional services on the basis of contingent fees.”
d. Incorrect See explanation for response c.

The correct answer is: Contingent fees should ordinarily be avoided.

127
Q

As defined in the APA’s General Guidelines for Providers of Psychological Services and the CPA’s Practice Guidelines for Providers of Psychological Services, “psychological services” include all of the following except?
Select one:

A. the conduct of scientific research
B. program development
C. consultation related to assessment and intervention
D. supervision of psychological services

A

The APA and CPA guidelines are both very specific about the activities that constitute “psychological services.”

a. CORRECT The conduct of scientific research is not included in the definition of psychological services.
b. Incorrect Program development is included.
c. Incorrect Consultation is included.
d. Incorrect Supervision is included.

The correct answer is: the conduct of scientific research

128
Q

A psychologist realizes that one of her therapy clients is a co-owner of a company that just hired her as a consultant. To be consistent with ethical requirements, the psychologist should:
Select one:

A. remedy the situation in a way that takes the client’s best interests into account.
B. continue in both roles but discuss any problems when they occur.
C. terminate her role of therapist or consultant.
D. terminate her role of consultant.

A

This is a difficult question to answer since none of the answers is incorrect. However, the best answer is the one that comes closest to the actual language of the ethical guidelines.

a. CORRECT The provisions of the APA and CPA ethical guidelines are somewhat ambiguous with regard to this issue. However, Standard 3.05(b) of the APA’s Ethics Code states that, when a multiple relationship occurs, “the psychologist takes reasonable steps to resolve it with due regard for the best interests of the affected person and maximal compliance with the Ethics Code”; and the Values Statement of Principle III of the Canadian Code of Ethics contains similar language.
b. Incorrect See explanation for response a.
c. Incorrect See explanation for response a.
d. Incorrect See explanation for response a.

The correct answer is: remedy the situation in a way that takes the client’s best interests into account.

129
Q

The research has found that therapists who have had sexual relations with their therapy clients are:
Select one:

A. younger and less experienced as psychotherapists than are those who have not had sexual relations with clients.
B. more likely than those who have not had sexual relations with clients to have been sexually involved with a supervisor, instructor, or their own therapist.
C. likely to say that the sexual relationship had a positive impact on the therapeutic relationship.
D. more likely than those who have not had sexual relations with clients to have had other non-sexual dual relationships with their clients.

A

Surveys of psychologists have found that those who report sexual relations with clients share some characteristics.
a. Incorrect Therapists who become sexually involved with clients tend to be older than those who have not, and professional experience has not been linked to sexual involvement with clients.

b. Incorrect D. H. Lamb, S. J. Catanzaro, and A. S. Moorman found that there was no statistically significant association between having a sexual relationship with a client and having had a sexual relationship with a supervisor, instructor, or therapist [Psychologists reflect on their sexual relationships with clients, supervisees, and students: Occurrence, impact, rationales, and collegial intervention, Professional Psychology: Research and Practice, 34(1), 102-107, 2003].
c. Incorrect In an earlier survey, Lamb and Catanzaro found that psychologists who reported having sexual relations with therapy clients were more likely to say the relationship had a negative (versus positive) impact [Sexual and nonsexual boundary violations involving psychologists, clients, supervisees, and students: Implications for professional practice, Professional Psychology: Research and Practice, 29(5), 498-503, 1998].
d. CORRECT In their 1998 survey, Lamb and Catanzaro found that psychologists who reported sexual relations with clients were also more likely to report having been involved in non-sexual dual relationships than were psychologists who had not had sexual relations with clients.

The correct answer is: more likely than those who have not had sexual relations with clients to have had other non-sexual dual relationships with their clients.

130
Q

Within the context of sexual harassment, “quid pro quo” refers to:
Select one:

A. a situation in which an employees response to sexual demands impacts his/her job.
B. the creation of a “hostile environment” as a result of sexually offensive conduct.
C. the judgment of a “reasonable woman” that an act is offensive.
D. overt (explicit) sexual conduct.

A

Quid pro quo is one of two legally recognized types of sexual harassment. Sexual harassment is addressed in Standard 3.02 of the APA’s Ethics Code, and the types of harassment are described in the Ethics Manual in the written study materials.

a. CORRECT Quid pro quo sexual harassment occurs when a person’s submission to or rejection of sexual advances affects his/her employment status. (Hostile environment sexual harassment is the other legally recognized type of sexual harassment.)
b. Incorrect See explanation for response a.
c. Incorrect See explanation for response a.
d. Incorrect See explanation for response a.

The correct answer is: a situation in which an employees response to sexual demands impacts his/her job.

131
Q

You have been seeing Malcolm M. for several months and believe that the original issues that Malcolm came to therapy for have been successfully resolved. When you bring up the topic of termination with Malcolm, he says that he wants to continue therapy. In subsequent sessions, you and Malcolm are unable to come up with additional treatment goals. As an ethical psychologist, your best course of action would be to:
Select one:

A. tell Malcolm that you are ethically required to terminate therapy and reassure him that he’ll be okay.
B. provide Malcolm with a brief period of pretermination counseling and discuss the reasons why termination is appropriate.
C. establish a contract with Malcolm for a maximum of three more months of therapy.
D. continue seeing Malcolm until he feels he is ready to stop seeing you.

A

Ethical guidelines are very clear about situations in which a therapist believes that it is the appropriate time to terminate therapy.

a. Incorrect See explanation for response b.
b. CORRECT Of the responses given, this is most consistent with ethical guidelines. Standard 10.10 of the APA’s Ethics Code requires psychologists to terminate therapy when it is “reasonably clear that the client/patient no longer needs the service” but to do so only after providing the appropriate pretermination counseling. Principles III.31 and 37 of the Canadian Code of Ethics contain similar language.
c. Incorrect See explanation for response b.
d. Incorrect See explanation for response b.

The correct answer is: provide Malcolm with a brief period of pretermination counseling and discuss the reasons why termination is appropriate.

132
Q

You are subpoenaed to appear at a deposition to testify about a former client of yours. During the deposition, the attorney for the opposing party asks you to provide confidential client information. You should:
Select one:

A. refuse to provide the information without a court order.
B. assert the privilege unless the client has signed a waiver.
C. provide the information if you believe it is pertinent to the case.
D. provide the information since privilege is waived in this situation.

A

Confidential client information should be revealed in a legal proceeding only when the client has signed a waiver or when the court has issued an order indicating that privilege is waived.

a. Incorrect You may provide the information if the client has signed a waiver.
b. CORRECT If the client’s attorney is present, you may ask him or her if you should answer the question. However, if the attorney is not present, you should not provide the information without a waiver from the client but, instead, should assert the privilege on behalf of the client.
c. Incorrect See explanation for response c.
d. Incorrect See explanation for response c.

The correct answer is: assert the privilege unless the client has signed a waiver.

133
Q

You believe that one of your clients is exhibiting symptoms of PTSD. A new test for PTSD has been developed by a psychologist at the local university, and you would like to use the test to evaluate the client. However, the test is currently labeled “For Research Purposes Only.” You should:
Select one:

A. get permission to use the test from the psychologist who developed it.
B. get an informed consent from the client before administering the test.
C. make sure that you combine test results with other data you have collected.
D. not use the test.

A

Ethical guidelines require that tests be used only for the purposes for which they have been validated.

a. Incorrect See explanation for response d.
b. Incorrect See explanation for response d.
c. Incorrect See explanation for response d.
d. CORRECT “For research purposes only” implies that this test has not been validated for clinical use and, therefore, should not be used for that purpose.

The correct answer is: not use the test.

134
Q

Which of the following is not an element of informed consent?
Select one:

A. credibility
B. comprehension
C. voluntariness
D. competence

A

Legally, three conditions are required for a consent to be truly “informed.” Note that this question is asking for the element that is NOT required.

a. CORRECT In order for a consent to be informed, the individual must have adequate information (knowledge) and be able to comprehend that information, must give consent voluntarily (without coercion), and must have the competence (capacity) to make rational decisions. Credibility is not one of the conditions for consent. Additional information on informed consent is provided in the Ethics Manual in the written study materials.
b. Incorrect See explanation for response a.
c. Incorrect See explanation for response a.
d. Incorrect See explanation for response a.

The correct answer is: credibility

135
Q

Psychologists are most likely to identify which of the following as the most frequently “ethically troubling issue” that they encounter in their professional practice?
Select one:

A. conduct of colleagues
B. confidentiality
C. treatment termination
D. sexual issues

A

The issues listed in this answer were all identified as “ethically troubling issues” in a survey of psychologists by K. S. Pope and V. A.Vetter [Ethical dilemmas encountered by members of the American Psychological Association: A national survey, American Psychologist, 47(3), 397-411, 1992].

a. Incorrect See explanation for response b.
b. CORRECT Of the 703 ethically troubling issues cited by psychologists in the Pope and Vetter study, the largest number (128) were related to confidentiality. In contrast, conduct of colleagues, treatment termination, and sexual issues were cited, respectively, 29, 5, and 28 times.
c. Incorrect See explanation for response b.
d. Incorrect See explanation for response b.

The correct answer is: confidentiality

136
Q

The Examination for Professional Practice in Psychology (EPPP) is best described as a:
Select one:

A. predictor of performance as a professional psychologist.
B. measure of basic knowledge of psychology.
C. measure of basic competence.
D. method for protecting the profession of psychology.

A

The EPPP is a requirement for licensure in the United States and Canada.

a. Incorrect See explanation for response b.
b. CORRECT The Association of State and Provincial Psychology Boards (ASPPB) describes the EPPP as a measure of “the knowledge that should have been acquired by any candidate who is seeking licensure to practice psychology.”
c. Incorrect See explanation for response b.
d. Incorrect See explanation for response b.

The correct answer is: measure of basic knowledge of psychology.

137
Q

In a research study in which it is necessary to use deception, participants are not told the exact nature of the study prior to their participation. In terms of ethical requirements, in this situation, the investigator is obligated to:
Select one:

A. explain why participants were not informed in the presentation of his research findings.
B. explain the true nature of the study to all participants as soon as possible.
C. file a waiver of informed consent with the institutional review board prior to beginning the study.
D. do none of the above as long as the study is determined to be of significant scientific value.

A

The use of deception is not prohibited by ethical guidelines but, when it is used, it must be accompanied by certain actions. Guidelines for the use of deception are provided in Standard 8.07 of the APA’s Ethics Code and are discussed in the Ethics Manual in the written study materials.

a. Incorrect This is not required by the ethical guidelines.
b. CORRECT When deception is necessary, participants must be provided with an explanation of the purpose of the study as soon as possible.
c. Incorrect This is not required by ethical guidelines.
d. Incorrect This is incorrect since “significant value” does not preclude explaining the true purpose of the study to participants as soon as possible.

The correct answer is: explain the true nature of the study to all participants as soon as possible.

138
Q

A psychotherapist in private practice includes the following information on her business card: sliding scale fees, first session free, APA member. This is:
Select one:

A. unethical because of sliding scale fees.
B. unethical because of first session free.
C. unethical because of APA member.
D. ethical.

A

None of the information included in the psychologist”s business card is prohibited by ethical guidelines.

a. Incorrect See explanation for response d.
b. Incorrect See explanation for response d.
c. Incorrect See explanation for response d.
d. CORRECT It is not unethical to charge clients on a sliding scale, and it would only be unethical to provide the first session free if there was some underlying coercion involved. It is also acceptable to list oneself as an APA member as long as it is not done in a way that is misleading.

The correct answer is: ethical.

139
Q

A psychologist who already has an established therapeutic relationship with a child is asked to act as an expert witness in a divorce proceeding involving custody of the child. The psychologist’s best course of action would be to:
Select one:

A. decline the request to testify.
B. agree to testify only if she obtains consent from both parents.
C. agree to testify only if she can evaluate all of the involved parties.
D. agree to testify only if her familiarity with the child won’t bias her testimony.

A

The psychologist already has an established relationship with the child, which changes the situation in an important way.

a. CORRECT Since the psychologist has a therapeutic relationship with the child, it would be in the child’s best interests for the psychologist to refrain from testifying. Instead, the psychologist’s role should be to provide the child with continued support.
b. Incorrect Even if both parents consent to her testimony, the psychologist would be engaging in a multiple relationship which ordinarily should be avoided in custody cases. See Paragraph II.7 of the APA’s Guidelines for Child Custody Evaluations in Family Law Proceedings.
c. Incorrect This would be the best course of action when a psychologist does not have a previous relationship with any of the involved parties.
d. Incorrect One of the reasons that the psychologist should not testify is because it might be difficult to be objective.

The correct answer is: decline the request to testify.

140
Q

A colleague, a licensed psychologist, confides in you that he has just ended a sexual relationship with one of his clients. As he describes the client, you realize that she is someone you referred to him about 18 months ago. As an ethical psychologist, you should:
Select one:

A. file a complaint against the colleague immediately with the Ethics Committee.
B. file a complaint against the colleague with the Ethics Committee after informing him that you intend to do so.
C. discuss the matter further with the colleague to see what course of action he plans to take.
D. contact the client and discuss the matter with her.

A

Ethical violations by colleagues are addressed by Standards 1.04 and 1.05 of the APA’s Ethics Code and Principles II.40 and 41 of the Canadian Code of Ethics.

a. Incorrect Filing a complaint with the Ethics Committee would be unethical if it involved a breach of the client”s confidentiality.
b. Incorrect See explanation for responses a and c.
c. CORRECT Ethical guidelines require psychologists to address ethical violations by other psychologists but to do so in a way that does not violate the confidentiality of the involved client. Therefore, this is the best course of action of those presented in the answers to this question.
d. Incorrect The question does not say that you have a current professional relationship with the client and, even if you did, it probably would not be appropriate to confront her with this from both an ethical and clinical perspective.

The correct answer is: discuss the matter further with the colleague to see what course of action he plans to take.

141
Q

A psychology professor at a large university asks a graduate student to assist on a research project in a field of interest to the student. The student will be required to spend about 25 hours a week on the project and will be paid on an hourly basis. With regard to publication credit, the professor should do which of the following when preparing a journal article about the study?
Select one:

A. He should include the student as a co-author since the student is working more than half-time on the project.
B. He should note the student’s contribution in a footnote or introductory statement since the student is essentially an employee.
C. He does not need to mention the student’s contribution since the student is an employee.
D. He should acknowledge the student in a manner consistent with her contribution to the project.

A

Standard 8.12 of the APA’s Ethics Code and Principle III.7 of the Canadian Code of Ethics apply to the situation described in this question.

a. Incorrect This would be necessary only if the student made a major contribution to the project.
b. Incorrect This would be sufficient only if the student made a minor contribution to the project. The fact that the student is being paid does not preclude publication credit.
c. Incorrect The fact that the student is being paid for her participation in the research project does not preclude publication credit.
d. CORRECT This response is most consistent with ethical guidelines. Standard 8.12(b), for example, states that “principal authorship and other publication credits accurately reflect the relative scientific or professional contributions of the involved, regardless of their relative status.”

The correct answer is: He should acknowledge the student in a manner consistent with her contribution to the project.

142
Q

A psychologist is hired by the court to evaluate the two young children of a couple who are seeking a divorce. The psychologist’s job will be to evaluate both children and report his findings to the court along with any recommendations regarding custody he can make on the basis of his findings. Through her attorney, the mother learns that the children have both revealed information to the psychologist that presents her in an unfavorable light. The mother contacts the psychologist and says that, since her children are both minors, she is the holder of the privilege and is, therefore, instructing the psychologist not to report the unfavorable information to the court. The psychologist should:
Select one:

A. comply with the mother’s request to not reveal the unfavorable information since she is the holder of the privilege.
B. provide the court with information about the results of the evaluation as long as he believes his evaluation is nonbiased.
C. provide the court with the information he feels is in the best interests of the children.
D. refuse to testify unless court-ordered to do so.

A

In custody cases, a psychologist must always put the best interests of the child first.

a. Incorrect See explanation for response c.
b. Incorrect See explanation for response c.
c. CORRECT As noted by E. J. Kermani (Handbook of Psychiatry and the Law, Chicago, Year Book Medical Publ., Inc., 1989), the concept of privilege “specifies that the parents” right must yield to the best interest of the child” regardless of who requested the evaluation in a custody case. In other words, parents cannot claim privilege or refrain from waiving privilege in legal matters pertaining to custody.
d. Incorrect The psychologist was hired by the court, so he does not need a court order in order to testify.

The correct answer is: provide the court with the information he feels is in the best interests of the children.

143
Q

Ethical standards for varying professions differ with regard to their emphasis and perspective. As a psychologist working in a multidisciplinary setting, you should:
Select one:

A. only work with fellow psychologists when developing professional and ethical standards for psychological services.
B. cooperate with other professionals when developing professional and ethical standards for psychological services.
C. consult with your local ethics committee for permission to work with other professionals when developing professional and ethical standards for psychological services.
D. develop professional and ethical standards for mental health services that do not recognize professional differences.

A

Answer B is correct: Psychologists frequently consult with other professionals, such as medical doctors, lawyers, and teachers. It is imperative that they be aware that every profession has professional and ethical standards that must be maintained. General Principle B of APA’s Ethics Code states that “psychologists consult with, refer to, or cooperate with other professionals and institutions to the extent needed to serve the best interests of those with whom they work”; and the Values Statement of Principle IV of the Canadian Code of Ethics states that “in order to be responsible and accountable to society, and to contribute constructively to its ongoing development, psychologists need to be willing to work in partnership with others.” This answer is most consistent with these guidelines.

Answer A: It is unrealistic to presume that it is possible to limit decision-making to psychologists only. There are many situations, especially in a multidisciplinary setting, where consultation with other professionals is necessary.

Answer C: Psychologists do not need to obtain permission to work with other professionals.

Answer D: Each profession has different professional and ethical responsibilities, and it is not advisable (and often not possible) to homogenize the responsibilities of different professions.

The correct answer is: cooperate with other professionals when developing professional and ethical standards for psychological services.

144
Q

Dr. Ja-Hoon Jang is conducting a research study that involves interviewing junior high school students about their relationships with peers. Prior to beginning the study, Dr. Jang obtained informed consents from the parents of the students as well as assents from the students themselves. During the course of the study, one of the students says he doesn’t want to complete the interview. Dr. Jang should:
Select one:

A. tell the student that he cannot withdraw from the study since one of his parents has signed an informed consent.
B. tell the student that he cannot withdraw from the study since he (the student) agreed to participate at the beginning of the study.
C. contact the students parents to ask them to encourage their son to complete the interview.
D. allow the student to withdraw from the study.

A

This issue is addressed by Standard 8.02 of the APA’s Ethics Code and Principle I.24 of the Canadian Code of Ethics.

a. Incorrect See explanation for response d.
b. Incorrect See explanation for response d.
c. Incorrect See explanation for response d.
d. CORRECT Ethical guidelines require researchers to allow participants to withdraw at any time from a research study. This answer is most consistent with this requirement.

The correct answer is: allow the student to withdraw from the study.

145
Q

Sexual harassment:
Select one:

A. is explicitly prohibited by ethical guidelines.
B. is not explicitly prohibited by ethical guidelines but is implicitly prohibited by standards limiting multiple relationships.
C. is not explicitly prohibited by ethical guidelines but is implicitly prohibited by standards limiting sexual intimacies.
D. is not explicitly prohibited by ethical guidelines but is implicitly prohibited by standards requiring avoidance of harm.

A

Sexual harassment is explicitly addressed by Standard 3.02 of the APA’s Ethics Code and Principle I.4 of the Canadian Code of Ethics.

a. CORRECT Standard 3.02 of the Ethics Code states that “psychologists do not engage in sexual harassment”; and Principle I.4 of the Canadian Code of Ethics states that psychologists “abstain from all forms of harassment, including sexual harassment.”
b. Incorrect See explanation for response a.
c. Incorrect See explanation for response a.
d. Incorrect See explanation for response a.

The correct answer is: is explicitly prohibited by ethical guidelines.

146
Q

Which of the following best describes ethical requirements regarding sexual intimacies with supervisees?
Select one:

A. Ethical guidelines do not prohibit sexual relationships with supervisees.
B. Ethical guidelines prohibit sexual relationships with supervisees under any circumstances.
C. Ethical guidelines prohibit sexual relationships with current supervisees only when the relationship will impair the psychologists objectivity and effectiveness as a supervisor.
D. Ethical guidelines prohibit sexual relationships with supervisees over whom the psychologist has evaluative authority.

A

Being familiar with the exact language of the ethical guidelines would have allowed you to identify the correct response to this question.

a. Incorrect See explanation for response d.
b. Incorrect See explanation for response d.
c. Incorrect See explanation for response d.
d. CORRECT Standard 7.07 of the APA’s Ethics Code and Principle II.28 of the Canadian Code of Ethics prohibit psychologists from becoming involved in sexual relationships with students and supervisees over whom they have evaluative authority.

The correct answer is: Ethical guidelines prohibit sexual relationships with supervisees over whom the psychologist has evaluative authority.

147
Q

Dr. La-Keysha Leonard, a licensed psychologist, has been seeing a prison inmate in group therapy for several months and is asked by the parole board to evaluate the inmate to assist with their decision regarding his parole. As an ethical psychologist, Dr. Leonard should:
Select one:

A. agree to evaluate the inmate.
B. agree to evaluate the inmate only if she believes it is in the best interests of the inmate to do so.
C. agree to do so only if she is allowed to explain the purpose of the evaluation and the limits on confidentiality to the inmate.
D. refuse to evaluate the inmate for the purpose of parole.

A

The situation described in this question is addressed by general ethical guidelines for multiple relationships and in the APA’s Specialty Guidelines for Forensic Psychology.

a. Incorrect See explanation for response d.
b. Incorrect See explanation for response d.
c. Incorrect See explanation for response d.
d. CORRECT This answer is most consistent with the provisions of the APA’s Ethics Code which prohibit psychologists from becoming involved in multiple relationships that might impair their objectivity and the APA’s Specialty Guidelines for Forensic Psychology which warns psychologists about the need for caution in dual relationships and roles in forensic settings. (The actions described in answer a, b, and c do not constitute sufficient caution.)

The correct answer is: refuse to evaluate the inmate for the purpose of parole.

148
Q

Which of the following best describes ethical guidelines regarding the use of inducements to encourage individuals to participate in a research study?
Select one:

A. Financial and other inducements are acceptable only when they do not coerce participation in the research study.
B. Financial and other inducements are acceptable only when the risks for participating in the study are clearly described to potential participants.
C. Financial and other inducements are acceptable only when alternative methods for obtaining participants are unavailable.
D. Financial inducements are unacceptable under any circumstances.

A

The use of inducements to encourage participation in a research study is addressed by Standard 8.06 of the APA’s Ethics Code and Principles I.27 and III.32 of the Canadian Code of Ethics.

a. CORRECT This answer is most consistent with ethical guidelines. For example, Standard 8.06 of the Ethics Code states that “psychologists make reasonable efforts to avoid offering excessive or inappropriate financial or other inducements for research participation when such inducements are likely to coerce participation.”
b. Incorrect This answer does not accurately describe ethical guidelines regarding the use of inducements.
c. Incorrect This answer does not accurately describe ethical guidelines regarding the use of inducements.
d. Incorrect Financial and other inducements are acceptable in some circumstances.

The correct answer is: Financial and other inducements are acceptable only when they do not coerce participation in the research study.

149
Q

Dr. Brian Bertram, a licensed psychologist, is hired by the owner of a company to evaluate employees who are not doing well on the job to determine if they would benefit from counseling or additional training. In this situation, Dr. Bertram should:
Select one:

A. reveal the information obtained from each employee to the owner of the company since she is the client.
B. reveal information about each employee only after obtaining a signed consent from employees since they are the clients.
C. not discuss confidentiality with the employees since to do so might limit their willingness to reveal information to him.
D. inform each employee prior to the evaluation of the potential limits on confidentiality.

A

In the situation described in this question, the company does have a right to relevant information, but this does not free the psychologist from the obligation to inform employees about the limits of confidentiality and the company about what kinds of employee information will be revealed.

a. Incorrect See explanation for response d.
b. Incorrect See explanation for response d.
c. Incorrect See explanation for response d.
d. CORRECT Standard 3.07 of the APA’s Ethics Code states that “when psychologists agree to provide services to a person or entity at the request of a third party, psychologists attempt to clarify at the outset of the service the nature of the relationship with all individuals or organizations involved. This clarification includes … the fact that there may be limits to confidentiality.” Principle I.26 of the Canadian Code of Ethics contains similar language.

The correct answer is: inform each employee prior to the evaluation of the potential limits on confidentiality.

150
Q

If a psychologist acts as both a fact witness for the plaintiff and an expert witness for the court in a criminal trial, she has acted:
Select one:

A. unethically by accepting dual roles.
B. ethically as long as she did not have a prior relationship with the plaintiff.
C. ethically as long as she clarifies her roles with all parties.
D. ethically as long as she obtains a waiver from the court.

A

The adoption of multiple roles is addressed in the ethics codes published by the American and Canadian Psychological Associations and in the APA’s Specialty Guidelines for Forensic Psychology.

a. Incorrect See explanation for response c.
b. Incorrect See explanation for response c.
c. CORRECT According to ethical guidelines, accepting multiple roles may be acceptable as long as certain conditions are met - e.g., as long as the psychologist considers the risks and benefits of doing so for all involved parties and clarifies the nature of the multiples roles with all involved parties. See Standard 3.05 of the APA’s Ethics Code and Principle I.26 of the Canadian Code of Ethics) and Paragraph 4.02 of the Specialty Guidelines for Forensic Psychology.
d. Incorrect See explanation for response c.

The correct answer is: ethically as long as she clarifies her roles with all parties.

151
Q

Which of the following best describes the requirements of the privacy rule of the Health Insurance Portability and Accountability Act (HIPAA)?
Select one:

A. The privacy rule always takes precedence over state laws.
B. The privacy rule is superceded by state laws when those laws provide greater privacy protection.
C. The privacy rule is enforceable only in the absence of state and other relevant federal laws.
D. The privacy rule applies only to organizations that receive financial assistance from the federal government.

A

HIPAA’s privacy rule provides standards for protecting the privacy and security of health information that is transmitted electronically.

a. Incorrect See explanation for response b.
b. CORRECT The privacy rule provides minimum standards for access to and use of patient information and is superceded by other laws that provide patients with greater control over their health records.
c. Incorrect See explanation for response b.
d. Incorrect See explanation for response b.

The correct answer is: The privacy rule is superceded by state laws when those laws provide greater privacy protection.

152
Q

Dr. Melba Morrison, a licensed psychologist, conducts research and teaches several classes at a large university. In her social psychology classes, she usually covers only material that is consistent with the results of her own research. In terms of the ethical guidelines, this practice is:
Select one:

A. ethical since it is up to Dr. Morrison to decide what to cover in her classes.
B. ethical as long as Dr. Morrison provides information about her policy in the class syllabus.
C. ethically questionable but not addressed by ethical guidelines.
D. clearly unethical.
A

This issue is addressed by Standard 7.03 of the APA’s Ethics Code and Principle III.12 of the Canadian Code of Ethics.

a. Incorrect See explanation for response d.
b. Incorrect See explanation for response d.
c. Incorrect See explanation for response d.
d. CORRECT To be consistent with ethical requirements when acting as an instructor or trainer, psychologists must present information accurately and in an unbiased way. Therefore, Dr. Morrison is acting unethically by presenting only information that is consistent with the results of her own research.

The correct answer is: clearly unethical.

153
Q

At the end of her third therapy session with Dr. Beatrice Bramwell, a client reveals that he is also in therapy with another psychologist. As an ethical psychologist, Dr. Bramwell should:
Select one:

A. inform the client that he’ll have to make a choice in the near future about which therapist to continue seeing.
B. inform the client that it would be unethical to continue seeing him while he is receiving treatment from another psychologist.
C. call the other psychologist to inform him of the situation and obtain his permission to continue therapy with the client.
D. discuss the treatment issues relevant to this situation with the client during the next session.

A

This issue is explicitly addressed by Standard 10.04 of the APA’s Ethics Code and indirectly addressed by Principle II.18 of the Canadian Code of Ethics.

a. Incorrect See explanation for response d.
b. Incorrect See explanation for response d.
c. Incorrect See explanation for response d.
d. CORRECT In this situation, Dr. Bramwell would want to consider the client’s welfare and avoid duplicating services or working at cross purposes with the other psychologist. Consequently, of the answers given, this is the best one.

The correct answer is: discuss the treatment issues relevant to this situation with the client during the next session.

154
Q

Which of the following techniques would be most useful for evaluating the impact of an intervention on patients’ quality of life?
Select one:

A. cost-feasibility analysis
B. cost-minimization analysis
C. cost-utility analysis
D. cost-benefit analysis

A

Cost analysis is conducted to determine the optimal allocation of financial resources. Experts distinguish between the four methods of cost analysis listed in the answers to this question. Additional information on these methods is provided in the Ethics and Professional Issues chapter of the written study materials.

a. Incorrect Cost-feasibility analysis is used to determine the feasibility of an intervention based on the monetary and other resources it requires.
b. Incorrect As its name implies, cost-minimization analysis is conducted to identify the least costly option among several options that produces equivalent outcomes.
c. CORRECT A cost-utility analysis is conducted to compare the costs and patient outcomes of an intervention, with outcomes including patients’ DURATION AND QUALITY OF LIFE.
d. Incorrect As its name implies, a cost-benefit analysis is used to determine the relative costs and benefits of a particular intervention.

The correct answer is: cost-utility analysis

155
Q

When evaluating a defendant’s competence to stand trial, the focus of the evaluation is on the defendant’s:
Select one:

A. ability to distinguish right from wrong.
B. ability to comprehend the charges against him/her.
C. mental state at the time he/she committed the crime.
D. DSM-IV diagnosis.

A

The focus of an evaluation to determine a defendant’s competence to stand trial focuses on his/her current psycholegal abilities and impairments.

a. Incorrect See explanation for response b.
b. CORRECT An evaluation for competence to stand trial focuses on the defendant’s ability to comprehend the charges against him/her, to understand the trial process, to cooperate with his/her attorney, etc.
c. Incorrect See explanation for response b.
d. Incorrect See explanation for response b.

The correct answer is: ability to comprehend the charges against him/her.

156
Q

An insurance company requests that you fax the company confidential client information. Assuming that the client has signed a release for this information, you should:
Select one:

A. refuse to fax the requested documents.
B. make sure the documents are clearly marked as confidential.
C. make sure that all identifying information is removed or coded.
D. ask the company to verify that the documents have been received after they are faxed.

A

Ethical guidelines that address the confidentiality of client records are relevant to the situation described in this question. See, e.g., Standard 6.02 of the APA’s Ethics Code.

a. Incorrect It would not be necessary to refuse to fax the requested information unless doing so would breach client confidentiality.
b. Incorrect Marking the material “confidential” would not guarantee that client confidentiality would be maintained.
c. CORRECT You would not have control over who sees the documents once they’ve been transmitted, and this course of action would ensure that client information is kept confidential.
d. Incorrect This would not necessarily be adequate for ensuring that someone other than an authorized person has had access to the documents.

The correct answer is: make sure that all identifying information is removed or coded.

157
Q

Dr. Vic Vasquez, the director of a private counseling center, prepares and distributes a brochure describing the center’s services. The brochure is available to potential clients and other interested people, and its text includes a list of the center’s staff members and their credentials. Specifically, it indicates that five of the six members have doctoral degrees from accredited universities and that the staff member who does not have a degree is a “Ph.D. candidate.” The designation of the nondegreed staff member is:
Select one:

A. acceptable because it accurately reflects the staff members status.
B. acceptable because it was not meant to be deceptive.
C. acceptable because this issue is not addressed by ethical guidelines.
D. unacceptable because it is potentially misleading.

A

The situation described in this question is addressed by Standard 5.01 of the APA’s Ethics Code and Principle III.2 of the Canadian Code of Ethics.

a. Incorrect See explanation for response d.
b. Incorrect See explanation for response d.
c. Incorrect See explanation for response d.
d. CORRECT Ethical guidelines caution against presenting one’s credentials in a way that could be misleading. Dr. Vasquez should not have listed the staff member as a “Ph.D. candidate” because it is possible that many people will not understand what this means.

The correct answer is: unacceptable because it is potentially misleading.

158
Q

A psychologist’s behavior as a private citizen:
Select one:

A. can never serve as grounds for a charge of unethical conduct.
B. may serve as grounds for a charge of unethical conduct if it violates ethical guidelines.
C. may serve as grounds for a charge of unethical conduct if it compromises his/her professional responsibilities.
D. will serve as grounds for a charge of unethical conduct when it also violates the law.

A

A psychologist’s private behaviors are addressed in the Introduction and Principle C of the APA’s Ethics Code and the Preamble of the Canadian Code of Ethics.

a. Incorrect See explanation for response c.
b. Incorrect See explanation for response c.
c. CORRECT The Introduction to the APA’s Ethics Code states that the “purely private conduct of psychologists … is not within the purview of the Ethics Code.” However, Principle C (Professional and Scientific Responsibility) notes that when “psychologists’ conduct may compromise their professional responsibilities or reduce the public’s trust in psychology and psychologists,” personal misconduct may constitute a violation of the Ethics Code. Similar language is included in the Preamble of the Canadian Code of Ethics.
d. Incorrect A minor legal infraction (e.g., traffic violation) is not likely to be grounds for a charge of unethical conduct.

The correct answer is: may serve as grounds for a charge of unethical conduct if it compromises his/her professional responsibilities.

159
Q

Vicarious liability is most likely to be an issue when a psychologist is acting in the role of:

a. supervisor.
b. faculty member.
c. advocate.
d. mentor.

A

a. supervisor.

When a psychologist is liable for the actions of his/her supervisee or employee, this is referred to as vicarious liability.

160
Q

Of the cases processed by the APA’s Ethics Committee as the result of revocation of a psychologist’s license by a state licensing board, the most common underlying cause for the loss of licensure is which of the following?

a. nonsexual dual relationship
b. sexual misconduct
c. confidentiality
d. practicing outside the boundaries of one’s competence

A

b. sexual misconduct

Loss of licensure is the most frequent reason for complaints processed by the APA’s Ethics Committee, and sexual misconduct is the most common underlying cause of the loss of licensure. See, K. S. Pope and M. J. T. Vasquez, Ethics in psychotherapy and counseling, New York, Wiley, 2007.

161
Q

Dr. A. T. Tester, a licensed psychologist, is scheduled to testify on behalf of the prosecution in a criminal case. He receives a request from the defendant to act as a fact witness. As an ethical psychologist, Dr. Tester should:

a. explain to the defendant’s attorney that he cannot do so since this would constitute a dual role, which is prohibited by ethical standards.
b. agree to do so only if he is able to get permission from the defendant and the plaintiff (or their attorneys) or from the court.
c. agree to act as a fact witness only after clarifying his responsibilities and role expectations with all parties.
d. agree to act as a fact witness since this does not conflict with his other role.

A

C. agree to act as a fact witness only after clarifying his responsibilities and role expectations with all parties.

The Specialty Guidelines for Forensic Psychology and the Ethics Code both caution against dual (multiple) roles in forensic settings but notes that, in some cases, they are acceptable. However, as noted in the Ethics Code, when a psychologist becomes involved in a dual relationship, he/she should “clarify role expectations and the extent of confidentiality” at the outset and subsequently as changes occur.

162
Q

During a court-ordered evaluation of a defendant to determine her competence to stand
trial, she reveals information to you that confirms her guilt. You should:

a. use your discretion in determining what to include in the evaluation report.
b. include the information in the evaluation report since privilege is waived in this
situation.
c. include only information relevant to the defendant’s competence in the evaluation report.
d. not provide the court with any information from the evaluation until the defendant signs a release.

A

C. include only information relevant to the defendant’s competence in the evaluation report.

In this situation, the defendant is protected by laws that prohibit the use of evaluation data to
determine a defendant’s guilt without his/her consent.

163
Q

Which of the following best describes the provisions of the APA’s Ethics Code regarding pro bono services?

a. The Ethics Code advises against providing free services to clients.
b. The Ethics Code recommends providing free services to clients.
c. The Ethics Code requires psychologists to provide free services to clients-in-need.
d. The Ethics Code does not address providing free or low-cost services to clients.

A

B. The Ethics Code recommends providing free services to clients.

“Pro bono” services are provided for the public good without compensation. Although the term “pro bono” is not used in the Ethics Code, General Principle B (Fidelity and Responsibility) states that “psychologists strive to contribute a portion of their professional time for little or no compensation or personal advantage.” Because free or low-cost services are mentioned in the General Principles (which provide aspirational guidelines), this means that the Code recommends – but does not require – psychologists to provide these services.

164
Q

Which of the following best describes requirements regarding the acceptance of contingent fees as provided in the Specialty Guidelines for Forensic Psychology (APA, 2011)?

a. Psychologists may provide services to a party in a legal proceeding on the basis of contingent fees.
b. Psychologists may provide services to a party in a legal proceeding on the basis of contingent fees only when the amount of the fees is “reasonable and consistent with current standards.”
c. Psychologists should seek to avoid providing professional services to a party in a legal proceeding on the basis of contingent fees.
d. Psychologists are prohibited from providing professional services to a party in a legal proceeding on the basis of contingent fees only when doing so represents an unacceptable dual relationship.

A

c. Psychologists should seek to avoid providing professional services to a party in a legal proceeding on the basis of contingent fees.

This issue is addressed in Paragraph 5.02 of the Specialty Guidelines for Forensic Psychology (APA, 2011), which states that “forensic practitioners strive to avoid providing professional services on the basis of contingent fees.”

165
Q

A psychologist has received a subpoena from an attorney requesting that she testify in court about a former client and provide the court with records related to the client’s treatment. The psychologist does not have a release from the client to do so. The psychologist should:

a. testify and release the records as requested.
b. testify but refuse to release the records without a court order.
c. not appear in court without a signed release from the client.
d. appear in court and claim the privilege on behalf of the client.

A

d. appear in court and claim the privilege on behalf of the client.

Assuming that the subpoena is valid, the psychologist must appear as requested but should
assert the privilege on the client’s behalf.

166
Q

An expert witness is recognized by the court to be qualified to offer opinions on a specific issue. To qualify as an expert witness, a psychologist must:

a. have a valid professional license.
b. have a relevant doctoral degree from an accredited school.
c. have appropriate education, training, and experience.
d. be certified as a forensic psychologist by the ABPP.

A

c. have appropriate education, training, and experience.

Requirements for expert testimony are provided in state and federal law. For example,
Federal Rule of Evidence 702 states: “A witness who is qualified as an expert by knowledge, skill, experience, training, or education may testify in the form of an opinion or otherwise if: (a) the expert’s scientific, technical, or other specialized knowledge will help the trier of fact to understand the evidence or to determine a fact in issue; (b) the testimony is based on sufficient facts or data; (c) the testimony is the product of reliable principles and methods; and (d) the expert has reliably applied the principles and methods to the facts of the case.”

167
Q

Dr. Opt assists with hiring decisions at the mental health clinic where he works. He recommends that an applicant for a staff psychologist position not be considered because of an unresolved charge of sexual harassment that has been filed against her with the Ethics Committee. Dr. Opt’s recommendation about the psychologist is:

a. consistent with the provisions of the Ethics Code.
b. a violation of the provisions of the Ethics Code.
c. ethical as long as the psychologist is considered when she is acquitted of the charge.
d. not addressed by the Ethics Code.

A

b. a violation of the provisions of the Ethics Code.

This response is most consistent with Standard 1.08 (Unfair Discrimination Against Complainants and Respondents) of the Ethics Code, which states that “Psychologists do not deny persons employment, advancement, admissions to academic or other programs, tenure, or promotion, based solely upon their having made or their being the subject of an ethics complaint.”

168
Q

A licensed psychologist who provides individual and group therapy to adolescents and adults learns that she is HIV+. In terms of ethical requirements, the psychologist:

a. should refrain from initiating any professional activities that might be adversely affected by her medical condition.
b. should obtain supervision to ensure that her medical condition does not impair her ability to provide effective services to clients.
c. should inform her clients of her medical condition “as early as is feasible.”
d. is not obligated to take any special actions or precautions in this situation.

A

a. should refrain from initiating any professional activities that might be adversely affected by her medical condition.

Standard 2.06 requires psychologists to “refrain from initiating an activity when they know or should know there is a substantial likelihood that their personal problems will prevent them from performing their work-related activities in a competent manner.” Personal problems include emotional, social, health-related and other personal issues.

169
Q

A client you have been seeing in therapy for several sessions tells you that her former therapist made repeated sexual advances toward her and that is why she stopped seeing him. She also says that she doesn’t want you to say anything about it to anyone. The psychologist is someone you see frequently at professional meetings. As an ethical psychologist, you:

a. inform the client that you are ethically required to report the psychologist to the Ethics Committee or state licensing board.
b. attempt to resolve the issue informally by talking to the psychologist.
c. convince the client that it is in her best interests to file a report with the state licensing
board herself.
d. tell the client that you will maintain confidentiality.

A

d. tell the client that you will maintain confidentiality.

This response is most consistent with the provisions of Standards 1.04 and 1.05, which require psychologists to give priority to client confidentiality when deciding how to handle knowledge about unethical behavior by a colleague.

170
Q

Dr. S. Towne is the only mental health professional in Micropolis, population 912, and the next town is 105 miles away. She finds that some of her clients have problems that are outside her area of training. Dr. Towne should:

a. see the clients only if they are in a crisis situation.
b. use only interventions that she already has experience using.
c. refuse to see the clients until she receives adequate training.
d. obtain supervision or consultation by telephone.

A

d. obtain supervision or consultation by telephone.

Although the Ethics Code requires psychologists to provide only services within the boundaries of their competence, there are exceptions. It is, for instance, perfectly acceptable for psychologists to acquire new skills and to treat new problems – but only when appropriate supervision or consultation are obtained. This is a difficult situation, but response d is the best given the circumstances and the alternatives provided in the responses.

171
Q

During the initial session with a therapy client, the client tells Dr. Goodenough that she and her husband are having “marital problems” and that she thinks they may need couples counseling. As the client describes her husband, Dr. Goodenough realizes that she had an affair with him 15 years ago in graduate school, long before he married the client. Dr. Goodenough’s best course of action would be to:

a. make an appointment with the husband to discuss the matter with him before setting up a couples session.
b. see the wife in individual therapy and refer them to another therapist for marital counseling.
c. see the couple in therapy only if she feels her previous relationship will not interfere with her objectivity.
d. not see the wife or the couple in therapy and make a referral to another therapist.

A

d. not see the wife or the couple in therapy and make a referral to another therapist.

This is covered by Standard 10.07, which prohibits psychologists from providing therapy to former sexual partners. In addition, providing therapy to the wife only would violate Standard 3.06, which states that psychologists refrain from engaging in professional activities when “personal, scientific, professional, legal, financial, or other interests or relationships could reasonably be expected to (1) impair their objectivity, competence, or effectiveness in performing their functions as psychologists.”

172
Q

A colleague at the mental health clinic where you are employed tells you that one of his clients has admitted to sexually abusing his daughter. The colleague says he is not going to report the abuse since it was mild in nature and the client has expressed remorse, promised not to do it again, and is willing to continue therapy. In addition, he feels that reporting the abuse would be detrimental to the community since the client is a widely-known, well-respected person. You should:

a. attempt to discuss the matter further with the colleague and encourage him to make a report to the appropriate child abuse reporting authority but do not make a report yourself.
b. file a complaint against the colleague with APA or the state licensing board.
c. contact the appropriate child abuse reporting authority and provide them with
information about the abuse and with the colleague’s name.
d. contact the appropriate child abuse reporting authority and provide them with information about the abuse but refuse to give them the colleague’s name if they request it.

A

C. contact the appropriate child abuse reporting authority and provide them with information about the abuse and with the colleague’s name.

This issue is addressed by Leland Swenson in Psychology and the Law for the Helping Professions, Brooks/Cole, Pacific Grove, CA, 1997. He states that “Any required reporter who knows that another required reporter has not reported must report the evidence of abuse as well as identify the nonreporter” (p. 413).

173
Q

The police arrive at your office with an arrest warrant for one of your clients. They tell you that they need the client’s most recent address and telephone number. You should:

a. provide them with the information they request since they have a warrant.
b. make a copy of the warrant for your files before providing them with the information
they request.
c. tell them you will have to get a signed waiver from the client before you can release any information about him.
d. not give them any information about the client.

A

d. not give them any information about the client.

Before providing information about a client (including the fact that he/she is a client), the psychologist would have to be provided with a search warrant, court order, or signed release from the client.

174
Q

You have just evaluated a child at the request of his mother. The child’s mother and father are separated and have joint custody. You receive a call from the father who says that he and his new girlfriend will be joining the mother when you meet with her to discuss the results of the evaluation. You call the mother, but she says she doesn’t want the girlfriend to be present at the meeting or to have information about the evaluation. You should:

a. comply with the mother’s request since she is the one who paid for the evaluation.
b. explain to the mother that you have to do what the husband requests since they have
joint custody.
c. tell the father that you will meet with him and his girlfriend at another time.
d. meet with the mother and father either together or separately but don’t include the girlfriend.

A

D. meet with the mother and father either together or separately but don’t include the girlfriend.

This response is most consistent with requirements regarding confidentiality. Legally (and ethically), the child’s parents have a right to information about the child and test data should only be released to them. Although there might be circumstances when it would be appropriate to provide evaluation data to other interested parties, there is nothing in this question to suggest that this is the case in this situation.

175
Q

You are a psychologist in private practice. In response to your growing caseload, you decide to bring in a second psychologist. The psychologist will be provided with office space and some secretarial services, and you will charge him 50% of fees collected from the patients you refer to him. This is:

a. unethical because referral fees are prohibited by the Ethics Code.
b. unethical because 50% is exploitative.
c. ethical as long as 50% is standard in your area of practice.
d. ethical as long as the 50% reflects your actual expenses.

A

D. ethical as long as the 50% reflects your actual expenses.

Referral fees are not entirely prohibited but, in situations like this one, must reflect the costs
incurred by the person making the referral.

176
Q

A new client says that she wants to pay for her therapy sessions with cash and that she doesn’t want you to keep a record of the sessions or of her payments. If you agree to this arrangement:

a. you have acted ethically as long as you ascertain that her reasons for making this request are valid.
b. you have acted ethically since it is up to you to decide what to include in your records. c. you have acted ethically and legally as long as you include her payments as income on
your income tax forms.
d. you have acted unethically and possibly illegally.

A

D. you have acted unethically and possibly illegally.

The APA’s Ethics Code and Record Keeping Guidelines require psychologists to maintain adequate records. This not only protects the welfare of the client but also protects the practitioner. In addition, many state laws require that client records be maintained and indicate the minimum information that must be included in such records.

177
Q

Dr. S. Swapper is starting a program for adolescents who are experiencing peer problems. She tells the editor of the local newspaper that his adolescent daughter (who is unpopular with her peers) can attend the program for free if the editor writes an article about the program for the paper. This is:

a. ethical as long as the arrangement doesn’t compromise Dr. Swapper’s objectivity when working with the editor’s daughter.
b. ethical as long as Dr. Swapper doesn’t tell the editor what to put in the article.
c. unethical because the Ethics Code explicitly prohibits this type of arrangement.
d. ethically dubious but not covered by the Ethics Code.

A

C. unethical because the Ethics Code explicitly prohibits this type of arrangement.

Standard 5.02 explicitly prohibits psychologists from compensating employees of the press for publicity about the psychologist and his/her work in a news item.

178
Q

Requiring students in an introductory psychology class to participate in a university sponsored research project as a requirement for the class is ethical only when:

a. the research is conducted by or under the guidance of a faculty member.
b. the research exposes students to “minimal risk.”
c. students are given a choice of an alternative assignment.
d. students are made aware of the requirement prior to enrolling in the class.

A

C. students are given a choice of alternative assignments.

This answer is most consistent with the requirements of Standard 8.04(b) of the Ethics Code, which states that, “when research participation is a course requirement or an opportunity for extra credit, the prospective participant is given the choice of equitable alternative activities.”

179
Q

When a research study might cause participants pain, deception:

a. is prohibited if the pain is physical but not if it is psychological in nature.
b. is ordinarily prohibited whether it is physical or psychological in nature.
c. is acceptable only if no alternative methods for obtaining the data are available.
d. is acceptable only if it is justified by the study’s “prospective value.”

A

b. is ordinarily prohibited whether it is physical or psychological in nature.

This response is most consistent with Standard 8.07(b), which states that “Psychologists do not deceive prospective participants about research that is reasonably expected to cause physical pain or severe emotional distress.”

180
Q

Which of the following statements is most consistent with the requirements of the Ethics Code regarding publication credit for publications that are substantially based on a student’s doctoral dissertation?

a. Authorship credit must reflect each individual’s relative contribution to the research project.
b. Authorship credit must be “mutually agreed upon” by all individuals who contributed substantially to the project.
c. The student must be listed as first author except under “exceptional circumstances.”
d. The student must always be listed as first author.

A

C. The student must be listed as first author except under “exceptional circumstances.”

This answer is most consistent with Standard 8.12(c), which states that, “except under exceptional circumstances, a student is listed as principal author on any multiple-authored article that is substantially based on the student’s doctoral dissertation.”

181
Q

The Ethics Code requires that a psychologist discuss with a new therapy client the issue of fees and policy regarding the provision of information to an insurance company:

a. prior to the first consultation.
b. during the first session.
c. when the psychologist deems it appropriate.
d. as early as is feasible.

A

D. as early as is feasible.

This is addressed in Standard 10.01, which states that, “when obtaining informed consent to therapy as required in Standard 3.10, Informed Consent, psychologists inform clients/patients as early as is feasible in the therapeutic relationship about the nature and anticipated course of therapy, fees, involvement of third parties, and limits of confidentiality.” Similar language is included in Standard 6.04.

182
Q

A client who terminated therapy with Dr. Jones several months ago calls to request an appointment because she is now very depressed and is feeling suicidal. In the past couple of months, Dr. Jones has been reducing his practice because he is starting another business. The best course of action in this situation is for Dr. Jones:

a. to see the woman until the crisis has passed and then provide her with appropriate referrals.
b. to explain to the former client that he is no longer accepting clients and refer her to a colleague.
c. to provide the woman with several referrals and ask her to call him if none of those turn out to be satisfactory.
d. to provide the woman with several referrals but let her know that he is available by phone in case of an emergency.

A

A. to see the woman until the crisis has passed and then provide her with appropriate referrals.

This course of action best takes the client’s welfare into account and, therefore, is most consistent with the “spirit” of the Code (see, e.g., Standards 3.04 and 10.09).